Nclex prep 3 Qs

¡Supera tus tareas y exámenes ahora con Quizwiz!

The nurse reviews the clinical data of a term newborn one hour following birth ➢ Click to highlight the body systems in the assessment that requires follow-up Vital signs Pulse 184/minute; Temperature of 95.7°F; Respiratory rate 65/minute Head 33 cm head circumference; flat and soft fontanels; symmetrical in shape Eyes Positive corneal and pupillary reflex; negative red reflex Cardiovascular S1 and S2 heart tones present; regular rhythm; femoral pulses palpable Lungs Abdominal respirations; clear lung sounds bilaterally; absent cough reflex Neuromuscular Floppy, poor head control, hypotonic movements in all extremities Abdomen Cylindric in shape; no meconium stool; normoactive bowel sounds Skin Generalized cyanosis; vernix caseosa present in skin folds; lanugo present on back Cry Weak, jittery cry

Assessment findings requiring follow-up Vital signs: the pulse is high (normal 110-160/minute; may go to 180/minute when crying). The temperature of 95.7°F (35.3°C) is low (normal 36.5°-37°C [97.7°-98°F]). The infant has tachypnea (normal respiratory rate 40-60/minute). These findings strongly suggest cold stress. Neuromuscular: floppy, poor head control; hypotonic movement in all extremities is abnormal. The expected finding would be the extremities in some degree flexion with good muscle tone. The infant should be able to turn their head from side to side when prone. Hypotonia may suggest hypoglycemia which could be potentially caused by cold stress. Eyes: the eyes should have a positive pupillary and corneal reflex. It is normal for the lids to be edematous for 2 days after birth. The red reflex should be positive. The red reflex is the reflection of light on the vascular retina. The absence of the red reflex may indicate glaucoma, retinal abnormality, retinoblastoma, or cataracts. Skin: generalized cyanosis is a concerning finding for cold stress. The expected integument finding is bright red, puffy, smooth, and some facial edema. The presence of vernix caseosa (a white, oily substance that coats the term infant's body and is often found in the creases of the axillae and groin) and lanugo (fine, downy hair found especially on the forehead, cheeks, shoulders, and back) is a benign finding. Cry: the cry should be vigorous. A weak, jittery cry is concerning for hypoglycemia. Assessment findings within normal limits Head: the head circumference should be between 33-35 cm - the head circumference should be approximately 2 to 3 cm more than the chest circumference. The fontanels should be soft and flat. Cardiovascular: S1 and S2 heart tones should be auscultated without any murmur. The rhythm should be regular. Femoral pulses should be palpable. Lungs: respirations are chiefly abdominal. The breath sounds should be clear without any adventitious sounds. Respirations that are chiefly abdominal are expected. The cough reflex is absent at birth and will be present in 1-2 days. Abdomen: The abdomen should be cylindric in shape. The umbilical cord should have two arteries and one vein. The bowel sounds should be nor

The nurse is preparing a client who is prescribed continuous epidural analgesia. Which of the following pre-procedure prescriptions should the nurse be prepared to administer? A. Lactated Ringers B. Ondansetron C. Ketorolac D. Haloperidol

Choice A is correct. Epidural analgesia has a common adverse reaction of hypotension. It is routine for clients being prepared for epidural analgesia to receive a liter of isotonic fluids to preempt this adverse reaction. Choices B, C, and D are incorrect. While epidural analgesia may cause nausea and vomiting, it is not routine to administer pre-procedure antiemetics such as ondansetron. Ketorolac is an antiinflammatory indicated for mild to moderate pain. This is not a medication indicated for an epidural. Finally, haloperidol is a typical (first generation) antipsychotic and has no relevance to an epidural.

While reviewing fetal monitoring strips, the labor and delivery nurse notes that the reading is nonreassuring. What features of the fetal monitoring strip does the nurse characterize as nonreassuring? Select all that apply. Fetal heart rate less than 110 beats/minute. Increase in variability. Late decelerations Mild variable decelerations Early accelerations

Choices A and C are correct. A fetal heart rate less than 110 beats/minute or greater than 160 beats/minute is nonreassuring (Choice A). Late decelerations are an ominous sign, therefore, immediate interventions should be taken to improve the fetal heart rate; they are characteristic of a nonreassuring heart rate (Choice C). Choice B is incorrect. An increase in variability is a reassuring factor. A decrease in variability would be nonreassuring. Choice D is incorrect. Mild, variable decelerations are okay, only when the variable decelerations are severe are they nonreassuring. Choice E is incorrect. Early accelerations are a reassuring factor on a fetal monitoring strip.

The nurse is caring for a client who was prescribed a clear liquid diet. Which dietary items would be appropriate for the nurse to include? Select all that apply. sherbert chocolate pudding vanilla yogurt apple juice coffee with oat milk fat-free bouillon with added salt tomato juice clear hard candy gelatin hot tea with added sugar

A clear liquid diet is usually transparent (to light) dietary items that do not contain dairy or pulp. Items such as water, gelatin, fat-free bouillon, hot tea, apple juice, seltzer, lemonade, and ginger ale are acceptable. Clear hard candy is acceptable because it is a clear liquid when melted. Salt and sugar are food additives that are permitted. Incorrect responses A full liquid is the next step when the diet is advanced. This diet contains opaque liquids. A full-liquid diet usually contains pulp and dairy. For example, coffee is a clear liquid, whereas a coffee with creamer or milk is a full liquid. Items that are full liquid include sherbert, milkshakes, frozen yogurt, pudding, strained soups, and coffee with dairy (or nondairy alternatives such as oat milk).

The nurse has become aware of the following client situations. The nurse should first follow up with the client who: A. has an irregular pulse that is receiving treatment for atrial fibrillation. B. has pneumonia who had an increase in temperature to 102°F (39°C). C. is receiving nebulizer treatments for asthma that suddenly stops wheezing. D. has active pulmonary tuberculosis (TB) and refuses prescribed medications.

C. is receiving nebulizer treatments for asthma that suddenly stops wheezing. Rationale: essentially cessation should not be abrupt wheezing should titrate down slowly. if stops abruptly could mean that oxygenation has stopped as well.

The emergency department (ED) nurse cares for a client with diabetes mellitus (type one). Which assessment finding requires immediate follow-up? A. Pulse 112/minute B. Persistent nausea and vomiting C. Respiratory rate 21/minute D. Blood glucose 299 mg/dL

Choice A is correct. A complication associated with DKA is hypovolemic shock. The client having tachycardia is demonstrating early signs of this type of shock. The treatment modalities of DKA include fluid repletion and insulin administration. Considering the client's tachycardia, the nurse should initially administer the prescribed isotonic fluids to treat the significant fluid volume deficit. Choice B is incorrect. Persistent nausea and vomiting is a manifestation of gastroparesis, which may be found with DKA. The nurse must address this finding by administering prescribed antiemetics (metoclopramide) and isotonic fluids. However, this is not the most immediate concern because the client is demonstrating early manifestations of shock. Choice C is incorrect. Tachypnea is an expected finding of DKA. The tachypnea is an attempt for the client to remedy the acid-based imbalance by having the client blow off the excess CO2. Having the client blow off the excess CO2 can treat the acidosis. The nurse is not concerned about an expected finding. Choice D is incorrect. The client diagnosed DKA with hyperglycemia and ketones, which a urine or blood specimen may detect. Clinical hyperglycemia is a blood glucose is greater than 250 mg/dL. This is not a priority because the client has DKA, manifested by hyperglycemia, an expected finding.

The nurse is caring for a client who ingested a lethal dose of aspirin (ASA). Which assessment finding is most concerning? A. Pulmonary edema B. Tinnitus C. Nausea and vomiting D. Tachycardia

Choice A is correct. All of these manifestations are associated with an aspirin overdose. Pulmonary edema is the most concerning and is caused by a lung injury induced by aspirin. A treatment for aspirin overdose is an infusion of sodium bicarbonate to correct metabolic acidosis. During the infusion, the nurse must be sensitive to the potential lung injury caused by aspirin; thus, auscultating lung sounds and assessing for pulmonary edema will be essential. Manifestations of pulmonary edema include tachypnea, tachycardia, and crackles in the lung fields. Choices B, C, and D are incorrect. All of these clinical manifestations are associated with an aspirin overdose. Tinnitus (humming, buzzing, or ringing in the ear) may be transient and is not life-threatening. Nause and vomiting are expected and are concerning because they may lead to hypovolemia. Treatment is parenteral fluids and prescribed anti-emetics. Tachycardia is likely with ASA poisoning because of the electrolyte shift and fluid volume depletion. Cardiac monitoring is the standard of care for this diagnosis. Add'l Additional Info ✓ Aspirin (ASA) overdose is highly concerning because it causes many serious effects ✓ The Poison Control Center should always be consulted with ASA overdoses for guidance on the client's care ✓ Manifestations of ASA overdose include Tinnitus Nausea and vomiting Tachypnea Metabolic acidosis Respiratory alkalosis (they may be in a mixed state of both) Tachycardia Hypovolemia Life-threatening pulmonary edema ✓ Treatment includes correcting the acid-based imbalance with sodium bicarbonate infusion(s), activated charcoal may be given if the ingestion was within two hours of presentation, cardiac monitoring, parenteral fluid replacement to correct hypotension, and glucose replacement

A 28-year-old female presents to the obstetrics office, suspecting she may be pregnant. Which of the following would the nurse recognize as a presumptive sign of pregnancy? A. Amenorrhea B. Positive fetal cardiac activity on ultrasound C. Enlarged uterus D. Auscultation of fetal heart tones

Choice A is correct. Amenorrhea (absence of menstrual period) is a presumptive sign of pregnancy. Presumptive signs of pregnancy are symptoms and signs that the patient experiences. Presumptive signs may resemble pregnancy signs and symptoms but may also be caused by other etiologies. While amenorrhea is a presumptive sign of pregnancy, missing a period can also result from other conditions such as stress, hypothyroidism, and anorexia. Choices B and D are incorrect. A positive cardiac activity on ultrasound (Choice B) and auscultation of fetal heart tones (Choice D) would be a positive sign of pregnancy. Positive signs of pregnancy are signs that cannot, under any circumstances, be mistaken for other conditions. Positive signs confirm that the pregnancy has occurred. Choice C is incorrect. An enlarged uterus would be detected by the examiner and would be a probable sign of pregnancy. Probable pregnancy signs indicate pregnancy in most cases; however, there is still the chance they can be caused by conditions other than pregnancy. While an enlarged uterus is a probable pregnancy sign, other conditions such as uterine tumors, fibroids, and adenomyomas may also cause such a finding. Another example of a probable sign of pregnancy is a positive urine pregnancy test.

he nurse has become aware of the following client situations. The nurse should first see the client who is receiving A. chemotherapy via a peripherally inserted central catheter (PICC) and reports blistering at the site. B. a chemotherapy infusion and develops nausea and vomiting. C. oral chemotherapy and reports burning in their mouth while drinking orange juice. D. external beam radiation therapy (EBRT) and sitting with visitors in the family waiting room.

Choice A is correct. Chemotherapy via a PICC line (or any vascular access) requires frequent assessment for infusion site reactions such as stinging and burning at the infusion site. This could be an indication of thrombophlebitis. If these manifestations are reported, the nurse should pause the infusion to assess the IV site and its patency. This is a priority to intervene because if the nurse fails to act, severe vascular damage may develop. Choices B, C, and D are incorrect. Nausea and vomiting are common effects associated with chemotherapy. The nurse may minimize these symptoms administered prescribed antiemetics or corticosteroids before the infusion of the prescribed chemotherapy. Another measure the nurse can take is to provide meals and snacks without aromas. The client receiving oral chemotherapy reported mouth pain, most likely stomatitis. Stomatitis is a common effect associated with chemotherapy. Treatment includes sodium bicarbonate mouth rinses and prescribed oral lidocaine. The client should avoid consuming aggravating dietary items, including acid, spice, and rough edges. The client receiving external beam radiation therapy (EBRT) has no limitation in their ambulation. The client receiving brachytherapy is confined to their room (and bed). To prevent dislodgement of the radioactive source. add'l info ✓ For a client receiving external beam radiation, the client should be instructed not to wash off the markings ✓ The radiologist must approve soaps and lotions before use. Soaps and lotions used generally do not have fragrances, dyes, or alcohol ✓ Clothing over the radiated site should be soft and loose ✓ The radiated area should not be exposed to the sun and should be covered with clothing

A nurse is precepting a new graduate nurse. They are working with a client with numerous family members at the bedside. Once they exit the room, the nurse asks the new graduate nurse to define "family." The new graduate nurse is correct when they state: A. "A family is a group of people who care about each other and work together to accomplish common goals or overcome hurdles." B. "A family includes a man and a woman who are married and the children they have together." C. "In order to be considered family, you have to be related through blood, marriage, or adoption." D. "Although there may be extended family elsewhere, the people who live in someone's house are their family members."

Choice A is correct. Families consist of groups of emotionally connected individuals who function as a unit. Choice B is incorrect. A family consisting of a man, woman, and any children they may have together is known as a nuclear family. Choice C is incorrect. While some benefits require relation through marriage, blood connection, or adoption (i.e., health insurance benefits, tax credits, etc.), these requirements are not mandated to be considered as one's family member. Choice D is incorrect. Although one may reside with members of their immediate family, an individual may also choose to reside alone, with a pet, with a roommate, with a friend, or with a romantic partner. The physical proximity in which one lives with another individual does not affect whether that person is considered family.

The nurse at a gynecology clinic speaks with a 25-year-old, 32-week pregnant client. The nurse assesses the client and the client's laboratory results. Which of the following findings should most concern the nurse? Glucose present in the urine B. The client reports +1 pedal edema at the end of the day C. The client reports increased vaginal discharge D. A hemoglobin level of 14 g/dL

Choice A is correct. Glucose in the urine may indicate gestational diabetes, as up to half of female clients have glucose in their urine at some time during pregnancy. Glucose in the urine may mean that a pregnant client has gestational diabetes. The nurse should promptly alert the health care provider (HCP) to allow further assessment into the cause of the glucose present in the client's urine. Choice B is incorrect. As pregnancy progresses, clients will often report the presence of slight pedal edema. This pedal edema occurs due to the pressure of the growing fetus on the vena cava, thus reducing venous return from the lower extremities. Choice C is incorrect. Increased vaginal discharge commonly occurs during pregnancy, especially as the client progresses toward the third trimester. As the client nears term, there is increased vascularity in the vagina and perineum. The nurse should inquire whether the discharge is foul-smelling, bloody, and/or abnormally colored, as these characteristics would be cause for concern and would need to be reported to the health care provider (HCP) for further evaluation. Choice D is incorrect. A hemoglobin level of 14 g/dL is an acceptable level for a pregnant client. Normal hemoglobin levels in pregnant women range from 11.5 to 14 g/dL. During pregnancy, the amount of blood in the client's body increases by about 20-30%, increasing the supply of iron and vitamins needed to make hemoglobin. Factors such as dehydration as well as overhydration can affect the test results. Add'l info: During routine prenatal care, clients undergo testing for gestational diabetes between 24 and 28 weeks of pregnancy. Approximately 6-10% of women in the United States develop gestational diabetes. Testing for gestational diabetes initially includes a glucose challenge test. If the client's blood glucose level exceeds the acceptable test range during the glucose challenge test, the client will subsequently undergo follow-up glucose tolerance testing to confirm or rule out a gestational diabetes diagnosis.

The nurse in the postanesthesia care unit (PACU) cares for a client who had an appendectomy. Which of the following client assessments warrants immediate follow-up? The client A. has breath sounds that are high-pitched and crowing. B. reports incisional pain at a level of '5' on a scale of 0 (no pain) to 10 (severe pain). C. has a capillary blood glucose of 115 mg/dL [70-110 mg/dL] D. reports persistent nausea following the administration of an anti-emetic.

Choice A is correct. High-pitched crowing sounds are consistent with a client having stridor. Stridor is a concerning adventitious breath sound because it indicates upper airway narrowing. The nurse needs to immediately respond to this client and determine if they have anaphylaxis to an intraoperative medication or a mechanical obstruction. Choice B is incorrect. Incisional pain after an appendectomy is expected. Compared to adventitious breath sounds, this does not require the nurses' immediate attention. Choice C is incorrect. The glucose being marginally elevated may be expected after surgery. Following stressful events, such as the appendectomy, the adrenal glands secrete more steroids causing the blood glucose to increase. This is not a priority concern. Choice D is incorrect. Post-operative nausea and vomiting are common, especially for younger clients. However, this is not a priority concern. It is not uncommon for a client to require various antiemetics to obtain nausea and vomiting abatement. This is not a priority concern for the nurse compared to the client's adventitious breath sound. Add'l Info: Stridor is characterized by a high-pitched crowing sound ✓ Stridor may occur with airway obstruction resulting from tracheal or laryngeal spasm or edema, mucus in the airway, or blockage of the airway from edema or tongue relaxation ✓ Treatment is the underlying cause and if it is caused by anaphylaxis, immediate administration of intramuscular (IM) epinephrine is the absolute priority

The nurse notices unlicensed assistive personnel (UAP) passing by several call lights during the shift. What is the nurse's best initial action? A. Approach the UAP about the behavior. B. Report unsafe behavior to the charge nurse. C. File an incident report due to safety risk. D. Ask another UAP to help cover this UAP's patient load.

Choice A is correct. Ignoring call lights (or not responding in a timely manner) puts patients at increased risk of falls and injury. The chain of command says the nurse should address issues/conflicts with the peer (if another nurse) or subordinate (UAP), as long as the situation is not illegal or dangerous. This nurse should first address issues with the UAP to determine the reason for this behavior (i.e. negligence versus work overload) and collaborate to find a solution. If the interaction is not effective, the nurse would then bring the issue up the chain of command (charge nurse) to determine the next steps. Choice B is incorrect. The nurse should address the UAP first before reporting. If the behavior continues, the issue should be brought up the chain of command to determine if it is due to factors such as negligent behavior, alarm fatigue, or improper staffing. Choice C is incorrect. Filing an incident report would not be appropriate. Although patient safety is at risk, according to the given information, no incident/accident has occurred. Choice D is incorrect. Asking another UAP to cover additional clients does not address the problem and increasing the number of patients for the second UAP would put the safety of additional patients at risk. Assignments should be set by the unit charge nurse and based on patient acuity.

The nurse is caring for a client with venous thromboembolism who has developed heparin-induced thrombocytopenia. After discontinuing the heparin infusion, the nurse anticipates which prescription from the primary healthcare provider (PHCP)? A. Enoxaparin B. Dabigatran C. Ketorolac D. Epoetin alfa

Choice B is correct. Heparin-induced thrombocytopenia (HIT) may be a life-threatening complication of exposure to heparinoids. The treatment for HIT Is to discontinue exposure to the heparin product immediately and to continue the anticoagulation with a non-heparin product. Agents that may be safely used include apixaban, dabigatran, or rivaroxaban. Choices A, C, and D are incorrect. Enoxaparin would be contraindicated because it is a heparinoid. Ketorolac is an anti inflammatory and not indicated in the treatment of HIT. Epoetin alfa is a colony-stimulating factor used to promote the production of red cells. This medication is not indicated in the treatment of HIT.

The nurse cares for a client scheduled for spinal surgery in one hour. The nurse anticipates that the primary healthcare provider (PHCP) will prescribe A. gentamicin. B. enoxaparin. C. hydromorphone. D. cyclobenzaprine.

Choice A is correct. Preoperative antibiotics are prescribed and administered before surgery to reduce surgical site infections (SSI). Orthopedic surgery, such as spinal surgery, is common in which a preoperative antibiotic is indicated. Preoperative antibiotics are given within sixty minutes of the surgical incision and may be continued to be given up until 24 hours after surgery. Choice B is incorrect. Enoxaparin should not be given before or immediately after spinal surgery as the risk for a hematoma may cause paralysis because of spinal cord compression. Additionally, enoxaparin should be withheld preoperatively because of the risk of hemorrhage. Choice C is incorrect. Hydromorphone is a powerful opioid that may be given postoperatively to alleviate pain. This medication has no clinical indication immediately before spinal surgery. Choice D is incorrect. Cyclobenzaprine is a muscle relaxer. This medication may be given postoperatively to provide adjunctive pain control. This medication is not indicated immediately before spinal surgery. Add'L Info Preoperative antibiotics are given within one hour of the surgical incision ✓ Medications commonly prescribed as preoperative antibiotics include nafcillin, cefazolin, vancomycin, and gentamycin ✓ Preoperative antibiotics may be continued to be given up to 24 hours after the surgery

A pregnant client who is Rh-negative is ordered an indirect Coomb's test. The nurse understands that the purpose of this test is to determine if A. antibodies are present from previous exposure to Rh-positive blood. B. the amount of time that it takes for fetal blood to clot. C. the blood type, Rh factor, and antibody titer of the newborn D. the fetus has a risk of developing pernicious anemia later in life.

Choice A is correct. Rh-negative women should have an indirect Coombs' test to determine whether they are sensitized (have developed antibodies) as a result of previous exposure to Rh-positive blood. This testing is done at the first prenatal visit. Choices B, C, and D are incorrect. Coombs' testing (direct or indirect) does not measure clotting time (either maternal or fetus). The blood type, Rh factor, and antibody titer of the newborn are determined by the direct Coomb test. These tests do not determine the probability of pernicious anemia, as this anemia is predicated on the lack of vitamin B12.

The nurse is taking a sample of the fluid pulled from a nasogastric tube to ensure proper placement. The nurse will confirm appropriate placement of the NG tube if the stomach contents have a pH of: A. 3.4 B. 7 C. 5.9 D. 8

Choice A is correct. The nurse would believe that the NG tube is correctly placed if the aspirate shows a pH below 5.5. Stomach contents should be acidic ( a pH less than 5.5). Before medication or food administration, it is crucial always to verify the correct placement of the nasogastric tube ( NGT). The gold standard to verify tube placement is visualization on an x-ray. However, given the risks of radiation exposure with X-rays and delayed feeding, alternative options are often used to verify the tube placement before feeding or giving medications to the client. The most commonly used first-line verification method is measuring the pH of the NG tube aspirate to make sure it falls in line with that of gastric contents. Most guidelines recommend that the pH of an NGT aspirate should be ≤5.5 (acidic) to confirm proper placement. Choices B, C, and D are incorrect. A pH of 7 or 8 is alkaline ( Choices B and D). An alkaline pH ( >7.0) often indicates a lung aspirate ( respiratory tract) rather than gastric. If the pH of the aspirate is greater than 5.5, a chest x-ray must subsequently be ordered to evaluate the NGT placement. Similarly, a pH of 5.9 ( Choice C) is not low enough to be considered a normal finding for gastric contents and hence, not adequate to ensure a proper NG tube placement. The subsequent step is to obtain a chest x-ray since this pH is greater than 5.5.

The nurse has received the following prescriptions for newly admitted clients. The nurse should initially implement which of the following? See the image below. A. initiate intravenous fluids to a client with anorexia nervosa. B. administer venlafaxine to a client with persistent depressive disorder. C. consult the social worker to begin discharge planning for a client. D. obtain a blood sample to evaluate a client's lithium level.

Choice A is correct. The priority is to attend to a client's physiological needs. Initiating intravenous fluids for a client with anorexia nervosa prioritizes over the other prescriptions because of the condition's ability to cause dehydration and severe fluid and electrolyte disturbances. Choices B, C, and D are incorrect. Administering venlafaxine for a client with a chronic depressive disorder is not the priority. This is a chronic problem, and acute problems come first. Consulting with the social worker for discharge planning is a low-priority task and is akin to providing discharge teaching. Obtaining a blood sample to evaluate a client's lithium level does not prioritize over a client-ordered intravenous fluid for a circulation problem.

The nurse reviews the vital signs of a client admitted to the medical-surgical unit. The unlicensed assistive personnel (UAP) indicates that the client's blood pressure was obtained in the client's leg. The nurse should expect which change in the blood pressure when taken in the leg? A. Systolic pressure in the legs is usually higher by 10 to 40 mm Hg B. Systolic pressure in the legs is decreased by 10 to 40 mm Hg C. Diastolic pressure is the decreased by 10 to 40 mm Hg D. Diastolic pressure is higher by 10 to 40 mm Hg

Choice A is correct. When blood pressure is obtained in the leg, the systolic blood pressure is increased by up to 10 to 40 mm Hg compared to blood pressure obtained over the brachial artery. The higher SBP is due to the calcification in the distal arteries, which raises the SBP. DBP in the lower extremities is usually the same when compared to the upper extremities. Choices B, C, and D are incorrect. SBP is increased when it is obtained in the lower extremities; it is not decreased. DBP is unaffected when obtained in the lower extremities Add'l info Dressings, casts, IV catheters, arteriovenous fistulas or shunts can make the upper extremities inaccessible for BP measurement. If this is the case, obtain the BP in a lower extremity. Systolic pressure in the legs is usually higher by 10 to 40 mm Hg than in the brachial artery, but the diastolic pressure is the same.

The nurse is caring for a client immediately postoperative following a below-the-knee amputation. The nurse should take which priority action? A. Elevate the stump on a pillow B. Check the operative site for bleeding C. Obtain an order for a physical therapy order D. Demonstrate the use of incentive spirometry (IS)

Choice B is correct. A complication following an amputation is hemorrhage. An early assessment and action of the nurse is to monitor the client for hemorrhage, which may be evident on the bandage, or if the client has a drain, a large amount of bloody drainage may be apparent. Findings that may support that the client is hemorrhaging include tachycardia with later development of hypotension. Rather the amputation is traumatic or surgically performed; it is hemorrhage that is a concerning complication. Choices A, C, and D are incorrect. These are all appropriate actions the nurse should take regarding caring for a client with a below-the-knee amputation. They do not prioritize the client's potential hemorrhage, which is a significant concern postoperatively. A client's mobility will be limited, and physical therapy should be consulted to start exercises, including bed mobility. A trapeze should be added to the bed to promote mobility and condition the upper extremities. Depending on the physician's order, most below-the-knee amputations have the stump elevated for the first 24 hours to decrease postoperative edema. After this, the client may be prone for 20-30 minutes four times daily to prevent a flexion contracture. The client should be taught how to use the IS because this helps decrease hypostatic pneumonia, a risk because of the client's limited mobility.

The nurse is caring for a client with suspected placenta previa. The nurse anticipates a prescription for which diagnostic test to confirm this finding? A. Manual cervical exam B. Transvaginal ultrasound C. Contraction stress test D. Nonstress test

Choice B is correct. A transvaginal ultrasound is a gold standard in diagnosing placenta previa. A transvaginal ultrasound is more accurate than transabdominal ultrasound in determining the location of the placenta. Please recognize that the vagal ultrasound probe can safely be inserted a few inches from the cervix to capture the placenta alteration. Any advancement of the probe into the cervix could cause bleeding. However, if done according to the recommendations, the appropriate angle between the cervix and the vaginal ultrasound probe prevents the probe from accidentally moving into the cervical canal. Choices A, C, and D are incorrect. A manual cervical examination is contraindicated in placenta previa because this may trigger bleeding. A manual cervical exam attempts to invade the cervical canal directly, whereas a transvaginal ultrasound probe is inserted away from the cervix. While placenta previa is not an absolute contraindication for a contraction stress test, it raises the risk of bleeding. A nonstress test is safe and often performed for individuals with placenta previa. However, this test does not confirm placenta previa; instead, it evaluates fetal wellbeing.

A client requests to change rooms after overhearing that their roommate is positive for the human immunodeficiency virus (HIV). The nurse should take which appropriate action? A. Relocate the client to a private room B. Ask the client to elaborate on their concern C. Notify the risk manager of the request D. Place an additional divider in-between the two beds

Choice B is correct. Asking the client to elaborate on their concern is the most logical and therapeutic action. The client is likely misinformed about the disease transmission of HIV, and the nurse should encourage the client to verbalize their concerns. It also is appropriate for the nurse to respond to any misconceptions the client may have with compassion and facts. Choices A, C, and D are incorrect. These actions are incorrect. Based on the information provided, the client should not be relocated to a private room. Additionally, if the client were to be relocated, a private room would not be necessary, as another semi-Private room would be appropriate. The risk manager does not need to be notified of the request. This request would be irrelevant to a risk manager. Finally, an additional divider is unnecessary because this would further fuel the hysteria surrounding HIV. A client with HIV can be roomed with another individual, and standard precautions should be utilized.

The nurse manager regularly performs chart audits and room inspections in the unit. She tells the staff to address the unit's deficiencies during a meeting. Which concept of management is the nurse manager displaying? A. Benchmarking B. Continuous Quality Improvement C. Performance Improvement D. Quality Management

Choice B is correct. Continuous quality improvement continually assesses and evaluates the effectiveness of client care. Choice A is incorrect. In Benchmarking, the nurse manager compares best practices from top hospitals to her unit and adapts the best unit's methods to improve unit performance. Choice C is incorrect. This establishes a system of formal evaluation for job performance and recommends ways to improve performance as well as promote professional growth. Choice D is incorrect. Quality management is the act of overseeing all activities and tasks needed to maintain a desired level of excellence. This includes the determination of a quality policy, creating and implementing quality planning and assurance, as well as quality control/improvement.

The nurse is performing a home visit to the parents of a newborn. The nurse witnesses the father's direct eye contact with the infant and shows affection. The nurse should document this finding as A. binding-in. B. engrossment. C. entrainment. D. detachment.

Choice B is correct. Engrossment is used to describe the initial paternal bonding. This is characterized by the father having a face-to-face fascination with the newborn and the ability for the father to start the bonding process. Choices A, C, and D are incorrect. Binding-in (also known as claiming) is a standard psychosocial process where the mother links the newborn's facial features to other family members. Entrainment is when the newborn moves rhythmically in response to adult speech. Detachment is unexpected and occurs when the bonding process fails and either the infant or parent has no bond with the other. Add'l info Engrossment is a positive finding in the initial bonding phase between the infant and the father. Engrossment is reinforced when the infant awakens and responds to the father's voice.

The nurse is caring for a client in labor who just received epidural analgesia. The nurse should monitor the client for which adverse effects? A. Hypertension B. Bladder distention C. Hypothermia D. Precipitous labor

Choice B is correct. Epidural analgesia may cause bladder distention. Bladder distention may cause pain that remains after initiation of the block and may interfere with fetal descent in labor. Choices A, C, and D are incorrect. Epidural analgesia is known to cause hypotension; thus, it is routine for a client to receive a preprocedural fluid bolus of isotonic lactated ringers. Hypothermia is unlikely with epidural placement. The fever associated with epidural analgesia is usually not caused by infection but may result from reduced hyperventilation and decreased heat dissipation. Epidural analgesia commonly slows the progression of the second stage of labor because it relaxes pelvic muscles.

The nurse is formulating a lesson plan for a presentation on teenage pregnancy for a local high school. To most effectively determine which topics to cover, the nurse must understand that: A. There is a low risk of complications in teen pregnancies B. Teenage pregnancies are commonly denied and concealed in the early stages C. Teens are using contraceptives at a decreased rate D. Teenage pregnancies are planned by teenagers as a form of rebellion

Choice B is correct. In the early stages of pregnancy, adolescent clients often deny or conceal their pregnancy. Following confirmation of a positive pregnancy test, common reactions include ambivalence, shock, fear, or apparent apathy. The nurse must emphasize the significance of early prenatal care to prevent complications in the adolescent's pregnancy. Although some clients may want to take some time before notifying their parent(s)/caregiver(s), prolonged denial or concealment of the pregnancy will lead to delayed prenatal care and related complications. The nurse should be aware that nonjudgmental nursing support is critical throughout this sensitive time for the safety of the teen and her pregnancy. Choice A is incorrect. Teen pregnancies are at an increased risk of experiencing a variety of complications. Medical concerns of adolescent pregnancies include prolonged labor, gestational diabetes, anemia, preeclampsia, low platelet syndrome, low-birth-weight infants, and increased infant mortality. Common causes of complications include the teenager's smaller stature and related incomplete growth process, poor nutritional intake, lack of resources, etc. Choice C is incorrect. Adolescent use of contraceptives (long-acting reversible contraceptives, intrauterine devices, birth control implants, etc.) has steadily increased since 2007, contributing to the declining risk of teen pregnancy. Choice D is incorrect. Most teen pregnancies are unplanned pregnancies. Although rebellion in teenagers is a common occurrence, teenage pregnancy is not a part of those rebellious acts.

The nurse is caring for a client with chronic obstructive pulmonary disease (COPD) with a chronically increased red blood cell count (RBC). The nurse understands that this finding is likely from: A. Increased carbon dioxide levels B. Low blood oxygen levels C. Insensible water loss D. Decreased fluid intake

Choice B is correct. Polycythemia is a condition with increased red blood cells in the blood. Low blood oxygen levels, a clinical feature associated with COPD, cause the kidneys to respond by releasing erythropoietin (EPO), which stimulates red blood cell production. The red blood cell count is elevated to compensate for hypoxia or low oxygen levels. More cells are available to carry and deliver the maximum amount of oxygen. Choices A, C, and D are incorrect. Hypercapnia, which is too much carbon dioxide in the blood, would not contribute to an increase in the red blood cell count (RBC) (choice A). Both insensible fluid loss and decreased fluid intake would result in dehydration. While dehydration (caused by sensible or insensible fluid loss) would cause hemoconcentration, dehydration is not a clinical finding linked to COPD. Moreover, dehydration causes transient hemoconcentration (relative/ spurious polycythemia) but not an actual, chronic increase in the RBC mass (choices C and D).

While working in the newborn nursery, you are called to L&D and asked to assign the APGAR score after birth. When you evaluate the infant at 1 minute of life, you find the following: cyanotic trunk and extremities, HR is 30 bpm, slight withdrawal when you pinch her foot, floppy muscles, and RR is 10 and irregular. What APGAR score do you assign? A. 1 B. 3 C. 5 D. 7

Choice B is correct. The APGAR score is 3. The infant gets 0 points for blue skin color all over, 1 point for an HR below 100 bpm, 1 point for a minimal response to stimulation, 0 points for absent muscle tone, and 1 point for a slow and irregular respiratory rate. This APGAR score indicates severe distress; therefore, the baby needs immediate action. Choices A, C, and D are incorrect.

You are a nurse in the L&D department of the local hospital. You are caring for a newborn born at term with APGAR scores of 8 and 10. Before discharge from the hospital, you should ensure that the newborn has received: A. Hep A (hepatitis A) vaccine B. Hep B (hepatitis B) vaccine C. RV (Rotavirus) vaccine D. DTaP (diphtheria, tetanus, and pertussis) vaccine

Choice B is correct. The Hepatitis B vaccine is given in three doses; the first dose is administered at the time of birth, the second dose at two months, and the third dose at six months of age. The Centers for Disease Control and Prevention (CDC) makes recommendations for vaccines and reviews special situations in vaccinations. Choice A is incorrect. Hep A vaccine is not given until the child is one year old. Choice C is incorrect. Rotavirus vaccine is given in 2 or 3 doses. The first dose is given at 2 months. Choice D is incorrect. Finally, the DTaP vaccine is given in 5 treatments with the first dose administered at two months.

The nurse is caring for an older adult following a total hip arthroplasty. The nurse should anticipate a prescription for which postoperative medication? Select all that apply. Hydrocortisone Enoxaparin Metoprolol Furosemide Morphine

Choice B is correct. The client following surgery is at high risk for venous thromboembolism (VTE). Considering that this client is older and they had a musculoskeletal procedure, this significantly increases the risk for postoperative VTE. Thus, anticoagulants such as heparin or enoxaparin are prescribed to prevent this complication. Following this surgery, the older adult may experience pain treated with opioids or non-opioids. Morphine is an opioid medication that may be given orally or parenterally to provide pain control. The nurse should assess the client's blood pressure and respirations before administering morphine because morphine may cause respiratory depression and hypotension. Choices A, C, and D are incorrect. Older adults and clients undergoing musculoskeletal surgery are at high risk for postoperative VTE. Anticoagulants are often prescribed with mechanical prophylaxis (sequential compression devices) to prevent this complication. Furosemide is a diuretic and has no direct utility postoperatively following this type of surgery. A beta-blocker is often given preoperatively to prevent postoperative cardiac dysrhythmias. However, this medication is given preoperatively - not postoperatively. Hydrocortisone is a corticosteroid and has no direct relevance to managing this surgical procedure.

The client is diagnosed with acute kidney failure. Which of the following is an appropriate psychosocial problem for the nurse to include in the care plan? A. Imbalanced nutrition: less than body requirements related to altered metabolic state and dietary restrictions. B. Anxiety related to the disease process and uncertainty of prognosis. C. Excess fluid volume related to compromised regulatory mechanisms secondary to acute renal failure. D. Risk for infection related to invasive procedures and an altered immune response secondary to renal failure.

Choice B is correct. The focus of the question is the psychosocial problem. Of the options listed, anxiety is the only choice for an appropriate psychosocial issue. Choice A is incorrect. This patient would be at risk for imbalanced nutrition due to the renal failure diagnosis, but this would not be a psychosocial problem. Choice C is incorrect. This patient would be at risk for excess fluid volume due to the renal failure diagnosis, but this would not be a psychosocial problem. Choice D is incorrect. This patient would be at risk for infection due to the renal failure diagnosis. However, this is not a psychosocial issue.

The nurse is interviewing a client who is pregnant in her first trimester, and the client inquires about their gestational age. The nurse understands which assessment will provide the most accurate gestational age? A. Nägele's rule B. Ultrasonography C. Fundal height D. Human chorionic gonadotropin (HCG) levels

Choice B is correct. Ultrasound measurements taken early in pregnancy (before 20 weeks of gestation) can more accurately determine the gestational age. Choices A, C, and D are incorrect. Nägele's rule is not the most accurate way to determine a prenatal client's due date. Especially if the menstrual cycle is irregular. Nägele's rule involves subtracting 3 months from the date the LNMP began, adding 7 days, and then correcting the year, if appropriate. Human chorionic gonadotropin (hCG) is produced by the trophoblastic cells surrounding the developing embryo. The primary function of hCG in early pregnancy is to prevent deterioration of the corpus luteum so it can continue producing estrogen and progesterone until the placenta is sufficiently developed. The presence of this hormone produces a positive pregnancy test result but does not assist in the determination of EDD. Fundal height is an inexpensive and noninvasive method of evaluating fetal growth and confirming gestational age. It is measured from the top of the symphysis pubis, over the abdominal curve, to the top of the fundus. This assists in the determination of the approximate gestational age, not EDD.

While auscultating a client's bowel sounds, the nurse notes a swooshing sound to the left of the umbilical area. What would be the nurse's priority action? Correct A. Percuss over the area to assess for dullness B. Notify the primary healthcare provided (PHCP) C. Gently palpate the abdomen to assess for tenderness D. Ask the patient about recent bowel movements

Choice B is correct. Upon auscultation, the nurse should suspect this client is presenting with an abdominal aortic aneurysm (AAA) due to the bruit or swooshing sound. The nurse should immediately notify the patient's healthcare provider of this urgent situation. An AAA Rupture can occur spontaneously or with trauma. If the aneurysm bursts, it may cause life-threatening bleeding. The aneurysm should be assessed immediately to determine the need for surgical intervention. Choices A, C, and D are incorrect. If a bruit is heard upon auscultation of the abdominal aorta, the nurse should suspect an aneurysm and not perform percussion or palpation due to the risk of rupture. Asking the patient about bowel movements (choice D) would be appropriate for assessing the patient's gastrointestinal system but would not be the highest priority for a suspected aneurysm.

The nurse has just inserted an indwelling urinary catheter for a male client. The nurse plans on securing the catheter to the client's A. inner thigh. B. lower abdomen. C. outer thigh. D. medial thigh.

Choice B is correct. When securing an indwelling urinary catheter for a male, anchoring it to the lower abdomen (with the penis pointed upward) or upper thigh is appropriate. The catheter tubing should be secured to the lower abdomen or the upper thigh to prevent posterior urethral injury. Choices A, C, and D are incorrect. These anatomical locations are inappropriate for securing an indwelling catheter for a male. When securing an indwelling catheter for a female, it should be anchored to the inner thigh. Additional Info When securing an indwelling catheter, the following should occur - ✓ The catheter should be attached to a male's upper thigh (with the penis pointed upward) or lower abdomen. ✓ For a female, the catheter should be connected to the inner thigh. The catheter should be secured with an adhesive device. The device is typically gently removed with an alcohol swab to avoid a shearing injury to the skin.

The RN is caring for a patient with suspected meningitis. Which action would the nurse recognize as the highest priority immediately following a lumbar puncture procedure? A. Test for gag reflex return B. Elevate the head of the bed to 30 degrees C. Encourage oral fluid intake D. Assess patient for Brudzinski sign

Choice C is correct. A lumbar puncture (or spinal tap) procedure is used to obtain cerebrospinal fluid (CSF) to diagnose meningitis and identify the cause. The nurse would encourage oral fluid intake following this procedure to replace CSF volume and reduce the risk of spinal headaches. Choice A is incorrect. A lumbar puncture procedure would involve local anesthetic at the site of the lower spine but would not involve sedation that would affect the gag reflex. Choice B is incorrect. The patient should be positioned lying flat for several hours following the lumbar puncture procedure to reduce the risk of spinal fluid leakage and spinal headache. Choice D is incorrect. A positive Brudzinski sign indicates meningeal irritation and may be used to screen for meningitis, but would not be appropriate to perform after the lumbar puncture procedure.

Chronic pain is most effectively relieved when analgesics are administered in what manner? A. On a PRN basis B. Conservatively C. Around the clock D. Intramuscularly

Choice C is correct. Around the clock, doses of analgesics are more useful for the management of chronic pain. Choice A is incorrect. A PRN protocol is inadequate for patients experiencing chronic pain. Choice B is incorrect. Conservative protocols may prove ineffective. Choice D is incorrect. Intramuscular administration for pain management is not practical on a long-range basis for a patient with chronic pain.

The nurse caring for a patient with Guillain-Barre syndrome is gathering supplies to keep near the patient's bedside. The patient is experiencing paralysis up to his waist. Which of the following instruments is of the highest priority? A. Blood pressure cuff B. Pulse oximeter C. Intubation tray D. Stethoscope

Choice C is correct. Ascending paralysis is an attribute of Guillain-Barre syndrome; therefore, these patients are at risk for respiratory failure. An intubation tray should be kept near the patient's bedside. Choice A is incorrect. A blood pressure cuff is helpful to have on hand but not the highest priority in a patient with ascending paralysis related to Guillain-Barre syndrome. Choice B is incorrect. A pulse oximeter is helpful to have on hand but not the highest priority in a patient with ascending paralysis related to Guillain-Barre syndrome. Choice D is incorrect. A stethoscope is helpful to have on hand but not the highest priority in a patient with ascending paralysis related to Guillain-Barre syndrome.

The nurse cares for a client and receives a phone call from the laboratory department regarding a critical sodium level of 122 mEq/L(135-145 mEq/L). The nurse should take which initial action? A. Notify the primary healthcare provider B. Implement seizure precautions C. Read back the result for verification D. Recollect the laboratory specimen

Choice C is correct. Before the nurse should execute any action, the nurse should read back the result to ensure effective and safe communication. It is essential that this process is not skipped to avoid client identification errors. Choices A, B, and D are incorrect. All of these actions are plausible for a client with severe hyponatremia as this may induce seizure activity. Recollecting the specimen may be necessary if the results are not clinically congruent or if contamination is suspected. Additional Info When critical results are obtained, the nurse should clarify the results by reading back the result. This protects client safety by ensuring that the result is linked with accurate client identification.

The nurse is caring for a neonate with a decreased cardiac output. If noted in this patient, which of the following is not a sign of decreased cardiac output? A. Oliguria B. Difficulty breastfeeding C. Bradycardia D. Hypotension

Choice C is correct. Bradycardia is not a typical symptom of decreased cardiac output in neonates. Instead, a decreased cardiac output generally results in tachycardia as the heart pumps faster to compensate. Typical signs of decreased cardiac output in an infant include oliguria, difficulty feeding, hypotension, irritability, restlessness, pallor, and decreased distal pulses. Choice A is incorrect. Oliguria is an expected finding in an infant with a decreased cardiac output. As the kidneys are perfused less efficiently in an infant with decreased cardiac output, urination reduces or ceases altogether. Choice B is incorrect. Difficulty breastfeeding may be seen in infants with low cardiac output. Feeding is increasingly difficult for babies with poor circulation. Choice D is incorrect. Hypotension is an expected finding in an infant with low cardiac output. Normal cardiac output is required to keep blood pressure regulated.

A nurse receives laboratory results for several clients under her care. Which client result would the nurse report to the primary health care provider (PHCP) first? A. An elevated amylase result in a client diagnosed with acute pancreatitis B. An elevated white blood cell (WBC) count in a client with a septic leg wound C. A urinalysis positive for leukocytes and nitrites in a chemotherapy client D. A serum glucose of 235 mg/dL in a client with type 1 diabetes mellitus

Choice C is correct. Chemotherapy agents increase clients' risk of infection due to immune suppression, specifically by decreasing neutrophils. Neutropenia, a reduction in the blood neutrophil count, is common in chemotherapy clients. The client's risk of bacterial and fungal infections increases with worsening neutropenia. Furthermore, if a bacterial or fungal infection occurs, the infection's likelihood of spreading to other parts of the body increases (sepsis). Early antibiotic intervention may prevent sepsis. In a urinalysis, the presence of leukocytes and nitrites is indicative of a urinary tract infection. This result should alert the nurse regarding a potential urinary tract infection in this immunocompromised client, warranting the nurse to notify the PHCP of the result so a complete blood count (CBC) can be obtained and antibiotic therapy may be initiated immediately. Choice A is incorrect. An elevated amylase result in a client diagnosed with acute pancreatitis is an anticipated finding and would not warrant reporting the result to the PHCP. Choice B is incorrect. In a client diagnosed with a septic leg wound, an elevated white blood cell count (also known as leukocytosis) is an anticipated finding. Leukocytosis usually occurs in response to infection, trauma, or inflammation. Since this client is known to be septic, the leukocytosis is an expected finding and, therefore, does not warrant the nurse immediately reporting this lab result to the PHCP. Choice D is incorrect. The client's serum glucose level of 235 mg/dL is above the normal range of 70-110 mg/dL. However, this is a relatively common finding in clients with type I diabetes mellitus and does not necessitate immediate reporting to the PHCP. Learning Objective Recognize that chemotherapy increases the risk of neutropenia and infections. The nurse must report any evidence of infection in a chemotherapy client to the PHCP and prepare to initiate prescribed empiric antibiotic therapy promptly.

While assessing a newborn infant in the nursery, you observe bounding 3+ radial pulses and faint 1+ pedal pulses. You also notice that the feet are cold and pale, while the hands are warm and pink. Which cardiac defect do you suspect this infant has? A. Tetralogy of Fallot (TOF) B. Hypoplastic left heart syndrome C. Coarctation of the aorta (COA) D. Transposition of the great arteries

Choice C is correct. Coarctation of the aorta is a narrowing of the aorta near the ductus arteriosus. Because of this narrowing, there is increased blood flow to the upper extremities and decreased blood flow to the lower extremities. That causes the symptoms described in the question: bounding upper pulses, faint lower pulses, and overall better perfusion to the upper extremities. Choice A is incorrect. In the tetralogy of Fallot, four defects are combined - an overriding aorta, pulmonary stenosis, hypertrophy of the right ventricle, and a VSD. At birth, the nurse would appreciate a murmur and mild to severe cyanosis, depending on the case. The described symptoms do not fit the tetralogy of Fallot. Choice B is incorrect. In hypoplastic left heart syndrome, the left side of the heart is underdevelopment. The nurse would note cyanosis and murmur at birth, but the described symptoms do not fit hypoplastic left heart syndrome. Choice D is incorrect. In the transposition of the great arteries, the pulmonary artery leaves the left ventricle, and the aorta leaves the right ventricle. These infants are severely cyanotic at birth and need surgery early in life, but the described symptoms do not fit the transposition of the great arteries.

The nurse is teaching a client who has hypertension about the newly prescribed medication, furosemide. Which of the following should the nurse include in the teaching? A. Limit intake of bananas, cantaloupe, and potatoes. B. Avoid taking the medication with grapefruit juice. C. Take this medication in the early part of the day. D. A nagging cough can occur as a side effect of the medication.

Choice C is correct. Furosemide is a loop diuretic and may be indicated for conditions such as heart failure or hypertension. The client should be instructed to take this medication in the earlier part of the day to avoid nocturia. Choices A, B, and D are incorrect. The client should be encouraged to have a high potassium diet as this medication is potassium wasting. Thus, bananas, cantaloupe, and potatoes are encouraged. Grapefruit does not interact with furosemide as it does with medications such as verapamil. A nagging cough is a common effect associated with ACE inhibitors.

The nurse is caring for a pregnant client with heart disease undergoing labor. All of the following are appropriate nursing interventions, except: A. Attach the client to a cardiac monitor and place an external fetal monitor. B. Manage pain early in labor. C. Use controlled pushing efforts. D. Encourage ambulation.

Choice D is correct. Bed rest should be maintained to conserve energy and decrease cardiac stress. Choice A is incorrect. The client and the fetus should be monitored frequently to assess for fetal distress and cardiac stress. Attaching the client to a cardiac monitor and placing an external fetal monitor ensures that the staff closely follows them. Choice B is incorrect. Pain increases cardiac stress in labor. The client should have adequate pain control; an epidural might be prescribed to control pain. Choice C is incorrect. Controlled pushing efforts decrease cardiac stress and conserve the client's energy.

The nurse plans a staff development conference about intimate partner violence (IPV). Which of the following statements, if made by a participant, would indicate a correct understanding of the conference? A. "Nurses are responsible for screening select individuals for intimate partner violence." B. "The nurse should tell the client that information about IPV will not be shared with anyone else." C. "Men may not report abuse because of a stigma of being abused by a woman." D. "Physical injuries from IPV are usually overtly seen on the face and the hands."

Choice C is correct. IPV against men is overwhelmingly underreported because of the stigma, masculinity expectations, shame, and certain laws that may favor women. Men oftentimes may sustain physical injuries to the genitals, verbal devaluation, and unprovoked physical attacks while sleeping. The nurse needs to keep an open mind regarding IPV because men may be the victim of the abuse and are not always the perpetrator. Choices A, B, and D are incorrect. Nurses are responsible for screening all individuals (not select). Almost half of the women murdered by an IPV event visited a hospital within the previous two years of their deaths. Further supporting the need for an assessment of all individuals. Nurses must reinforce that all information shared about violence will be kept confidential but will be reported as required by law. Telling the client that the information will not be shared with anyone else is not entirely factual, as the nurse must report the event to authorities to share the pertinent details. The nurse should be complete when explaining to the client that their IPV concern will be kept confidential but could be shared with the treating physician and authorities. Physical injuries from IPV are often concealed under articles of clothes (missing hair under a hat, bruises to the torso and genitals). The nurse should remember that injuries are likely to be concealed, and permission should be obtained before an examination to avoid startling the client.

The nurse is preparing to remove an intrajugular central venous catheter. It would be appropriate to place the client in which position for this procedure? A. Reverse Trendelenburg B. Left lateral C. Trendelenburg D. High-Fowler's

Choice C is correct. Placing the client in a supine or Trendelenburg position while removing a central venous catheter would be appropriate. One of these two positions is acceptable to decrease the risk of air embolism. The client should not have their head elevated for this procedure because that would increase the risk of air embolism. Choices A, B, and D are incorrect. The positioning of a client is essential to avoiding an air embolism. Thus, having a client in high Fowler's, lateral, or reverse Trendenlenberg would be contraindicated. If a client experiences an air embolism, turning the client to the left-lateral decubitus position would be appropriate, but not for the procedure of removing a central line itself. Learning Objective Recognize the interventions to minimize the risk of air embolism while removing a central venous catheter. These include placing the client in a Trendelenburg or supine position. Add'l info While removing a central venous catheter, the client should be positioned supine or in Trendelenburg. The catheter should not be removed while the client is sitting up because this would increase the risk of air embolism because the atmospheric pressure is relatively higher than the intrathoracic pressure, and this propels the air into the catheter. To ensure that the intrathoracic pressure is higher than atmospheric pressure, position the client in either supine or Trendelenburg and have the client hold their breath or perform a Valsalva maneuver during removal.

The nurse in the nursery is caring for a 24-hour-old infant. The nurse suspects the infant of having pyloric stenosis. Which of the following manifestations would support this finding? A. Melena B. Currant jelly stools C. Projectile vomiting D. Steatorrhea

Choice C is correct. Projectile vomiting (i.e., vomiting accompanied by vigorous peristaltic waves) is typically associated with pyloric stenosis. Choice A is incorrect. Melena is the passage of black, tarry stools that contain denatured (digested) blood and suggests an upper gastrointestinal source of bleeding. Melena is not a manifestation of pyloric stenosis. Choice B is incorrect. Blood with mucus in the stool indicates an inflammatory or infectious condition, and currant jelly-like stools indicate vascular compromise, traditionally associated with intussusception. Choice D is incorrect. The definition of steatorrhea is an increase in fat excretion in the stools. Steatorrhea is one of the clinical features of fat malabsorption and is noted in many conditions (i.e., exocrine pancreatic insufficiency, celiac disease, etc.). An increase in the fat content of stools results in the production of pale, large-volume, malodorous, loose stools. Steatorrhea is not an associated manifestation of pyloric stenosis.

According to the American Nurse Association Code of Ethics, "liability with the performance of duties in a specific role" is: A. Accountability B. Authority C. Responsibility D. Delegation

Choice C is correct. Responsibility involves liability with the performance of duties in a specific role. Essentially, this means that when an LPN/LVN accepts an assignment, they also take responsibility for performing the task correctly. Accountability refers to the review of actions to determine if they were performed successfully. This means that the RN verifies that the LPN/LVN accepts responsibility for the task that is delegated to them. Authority in the delegation process means that the RN can legally transfer responsibility to another competent individual on the team. The RN also has the authority to complete assessments, plan/evaluate nursing care, and exercise nursing judgment in the course of care. Choices A, B, and D are incorrect. Accountability, authority, and trust are all aspects of the delegation process.

Which of the following maternal infections may increase the risk of developing congenital heart defects in the fetus? Correct A. Parainfluenza B. Adenovirus C. Rubella D. Measles

Choice C is correct. Rubella is a maternal infection that is known to increase the risk that the fetus will have a congenital heart defect. All mothers should be tested for rubella, and if found to be positive, should have a fetal echocardiogram performed to evaluate the fetus' heart more closely. Choice A is incorrect. Parainfluenza is not known to affect the risk for congenital heart disease. Choice B is incorrect. Adenovirus is not known to affect the risk for congenital heart disease. Choice D is incorrect. Measles is not known to affect the risk for congenital heart disease.

A 5-year-old child suspected of having attention-deficit/hyperactivity disorder (ADHD) is brought to the pediatric clinic for an evaluation. Which behavior, if observed by the clinic nurse, would support the suspicion of ADHD? A. Lethargy B. Preoccupation with body parts C. Poor verbal skills D. Short attention span

Choice D is correct. Children with ADHD exhibit short attention spans due to a variety of factors. Inattention tends to appear when a child is involved in tasks that require vigilance, rapid reaction time, visual and perceptual search, and systematic and sustained listening. Choice A is incorrect. Hyperactivity involves excessive motor activity. Children, particularly younger ones, may have trouble sitting quietly in environments where they are expected to do so (e.g., in school or church). Therefore, a child with ADHD would exhibit increased psychomotor activity, not lethargy. Choice B is incorrect. A child with ADHD is distracted by external factors and is very much distracted by external environmental stimuli. Choice C is incorrect. Poor verbal skills are not associated with ADHD.

A medical-surgical ward nurse is taking a floater shift in the ICU. The charge nurse would assign which patient to the nurse? A. A 2-hour post lung transplant client B. A client with a central venous pressure ( CVP) reading of 13 cm of water C. A client with pneumonia D. A client with Hantavirus pulmonary syndrome

Choice C is correct. The client with pneumonia is neither in immediate danger nor in complicated condition. The medical-surgical nurse can safely assess and administer medication to this client. This client can be assigned to the nurse. Choice A is incorrect. The client is still in critical condition and is prone to organ rejection. This client should be assigned to an experienced nurse. Choice B is incorrect. The client's increased central venous pressure ( CVP) reading indicates volume overload or right ventricular failure. This client needs to be assigned to a much more experienced nurse. Choice D is incorrect. Hantavirus pulmonary syndrome is a deadly disease with a specific treatment or cure. This client should be assigned to an experienced nurse. Test-taking strategy: Please note that the question is testing you about assignment/ delegation, not prioritization. While assigning a patient to a nurse, one should delegate it to the person trained to handle that task.

The patient presents to the emergency department with back pain and numbness in the extremities after experiencing a fall. The nurse assesses muscle flaccidity and hypotension. What is the nurse's highest priority regarding this patient? A. Assess for external bleeding B. Prepare the patient for intubation C. Stabilize the cervical spine D. Insert an 18g IV for fluid replacement

Choice C is correct. The highest priority action would be to stabilize the patient's cervical spine to prevent further damage and to preserve airway patency. Choice A is incorrect. The nurse should assess the patient for bleeding and other injuries due to the trauma, but it would not be the highest priority/first action. Choice B is incorrect. The patient may require intubation if the airway is not patent. Still, the nurse would first need to stabilize the spine and assess the airway before knowing whether intubation is appropriate. Choice D is incorrect. Inserting a large-gauge IV catheter would be an appropriate action for this patient, but not the highest priority. Fluid replacement would not be a more immediate need than the airway.

The nurse is assisting in the placement of an indwelling Foley catheter in a male patient. The nurse knows to inflate the balloon on the catheter at which step in the procedure? A. Upon meeting resistance B. As soon as urine is observed in the tubing C. After advancing to the point of bifurcation D. After fully advancing the length of the catheter

Choice C is correct. The nurse should inflate the balloon on the catheter once she reaches the point of bifurcation. This is achieved by slowly advancing the catheter, observing the tubing for urine to appear, and then continuing to advance to the point of bifurcation after urine is observed. This will ensure the balloon is in the bladder before the nurse inflates it. Choice A is incorrect. It is not appropriate to inflate the balloon on the catheter upon meeting resistance. In a male client, this could cause serious trauma to the urethra. The nurse must ensure that the catheter is fully inside the bladder before the balloon is inflated. Choice B is incorrect. It is not appropriate to inflate the balloon on the catheter as soon as urine is observed in the tubing. The catheter will not be fully in the bladder as soon as urine is observed and inflating the balloon at this point would cause trauma. Choice D is incorrect. It is not appropriate to fully advance the length of the catheter in every client. The length of the urethra and distance to the bladder will vary and therefore the catheter will be advanced in different lengths depending on the client. Fully advancing the catheter could result in an excessive length of the catheter sitting in the bladder, which can cause pain and irritation to the client.

The emergency department (ED) triage nurse is assigned to see the following clients. Which of the following clients requires the most rapid action in the ED? A. A travel blogger who needs tuberculosis testing after exposure to a person with TB during his trip. B. An elderly woman who has a history of a methicillin-resistant Staphylococcus aureus (MRSA) leg wound infection. C. A pregnant woman with a blister-like rash on the face who possibly has varicella. D. An infant with a runny nose and whose older brother has pertussis.

Choice C is correct. The primary responsibility of the triage nurse is to perform an initial nursing assessment and determine which patient(s) require immediate care or isolation. The triage nurse should be able to identify patients who pose a potential risk to others by being familiar with commonly occurring illnesses/infections. Emergency department nurses and triage nurses must be adept at prioritization. Prioritization refers to the concept of deciding which duties/clients require immediate attention and which ones could be delayed until later. None of the clients in the options above show any signs of unstable vitals. Therefore, the safety of the client and other clients takes priority. Chickenpox (Varicella) is transmitted airborne and can be easily transferred to other clients in the emergency unit. The pregnant woman with suspected Varicella rash (Choice C) should be isolated right away from other clients through placement in a negative-pressure room. Choice A is incorrect. The client who has been exposed to Tuberculosis (TB) does not place the other clients at risk for infection because he/she has no symptoms of active TB. Latent tuberculosis is not infectious by itself. Pulmonary/Cavitary disease can manifest with a productive cough and carries the highest risk of infection. Only such symptomatic TB patients should be placed in a negative-pressure room with airborne isolation precautions. Choices B and D are incorrect. Droplet precautions should be instituted for the client with possible pertussis. Contact isolation should be implemented for the client with a history of MRSA infection. But these two patients should be attended to after isolating the pregnant client with possible varicella because the risk of infectivity with the airborne transmission is much higher.

While working in the neonatal intensive care unit, the nurse assesses the client receiving continuous nasogastric feeding. In the gastrointestinal assessment, the nurse notes: Hypoactive bowel sounds The abdominal girth of 32 cm increased from 30 cm at the previous assessment. Soft abdomen, tender to palpation. No stool x 2 days. What is the priority nursing action? A. Notify the healthcare provider B. Continue to monitor C. Pause the infant's feeds D. Re-evaluate the abdominal girth at your next assessment

Choice C is correct. The priority nursing action is to pause the infant's feeds. Next, the nurse should notify the healthcare provider. Infants in the neonatal intensive care unit (NICU) have a high risk of developing necrotizing enterocolitis (NEC). One of the first signs of NEC is the inability of the infant to tolerate feedings. The feeding intolerance is evidenced by abdominal distension and bilious vomiting. This patient is showing potential signs of early necrotizing enterocolitis: hypoactive bowel sounds, increased abdominal girth, and no stool. One of the significant complications of NEC is bowel perforation. If the patient's bowel perforates, it is a medical emergency that can lead to sepsis and death. To reduce this risk, the priority nursing action is to pause the feeds to avoid further damage. Pausing the feeds is an independent nursing action in this urgent scenario, and after that, the nurse can proceed to notify the health care provider. Choices A, B, and D are incorrect. Notifying the health care provider, monitoring the patient, and reassessing the abdominal girth for worsening distension are all necessary actions. However, pausing the feeds to prevent further complications takes priority over the other listed actions. If the NEC is recognized in its early stages, feedings can be discontinued, the stomach can be decompressed by inserting an orogastric tube, and antibiotics may be started to address possible sepsis.

The nurse is preparing medications for the shift. Which of the following clients should the nurse prioritize for immediate medication administration? A. Digoxin to a client with atrial fibrillation B. Furosemide to a client with congestive heart failure C. Magnesium sulfate to a client with Torsades de pointes D. Labetalol to a client with a blood pressure of 160/100 mmHg

Choice C is correct. Torsades de pointes is an emergency because it is life-threatening and can progress to ventricular fibrillation and sudden cardiac death if not promptly treated. The nurse should immediately administer the prescribed magnesium sulfate to the client to prevent Torsades from degenerating into ventricular fibrillation. Choices B, A, and D are incorrect. All these medications are necessary for these clients and must be administered. However, the client with a more life-threatening condition (Torsades de pointes) should be prioritized. Furosemide is a loop diuretic used to treat congestive heart failure and edema. Digoxin is used to treat atrial fibrillation with the goal of controlling the heart rate and restoring sinus rhythm. However, some recent studies have shown increased mortality with chronic use of digoxin in atrial fibrillation. Labetalol is a beta-adrenergic and a selective alpha-1 adrenergic blocker. Therefore, it is widely used to treat arterial hypertension (from chronic to hypertensive crises). The client's blood pressure is high, but they are not exhibiting signs of target organ dysfunction (stroke, myocardial infarction, or heart failure) to suggest a hypertensive emergency. The client with Torsades de pointes should be attended to before the client with stable hypertension.

The nurse is caring for a client admitted with severe pre-eclampsia. It would be essential for the nurse to have which of the following items at the bedside? A. One liter of 0.9% saline B. Sterile gloves C. Portable ultrasound D. Suction equipment

Choice D is correct. A client with severe pre-eclampsia should be monitored closely for seizures which are the hallmark manifestation of eclampsia. The nurse should plan care involving seizure precautions at the bedside, including suction equipment, padded side rails, and oxygen. Choices A, B, and C are incorrect. Sterile gloves, portable ultrasound, and a liter of 0.9% saline would not be necessary to manage a client having a seizure directly related to pre-eclampsia. These tools would be helpful for other obstetric procedures but not for a severely pre-eclamptic client at risk of having a seizure. Add'l info Severe pre-eclampsia may require intensive care monitoring, depending on other factors. Nursing care for a client with severe pre-eclampsia includes: ➢ Appropriate safety equipment at the bedside, which includes seizure precautions. ➢ Frequent vital signs and blood pressure must be closely monitored as a hypertensive emergency may develop. ➢ Prescribed medication administration such as magnesium sulfate. ➢ Frequent fetal well-being assessment that includes continuous fetal heart rate monitoring.

The nurse is caring for a prenatal client with some vaginal bleeding. The nurse knows that this client could be experiencing a spontaneous abortion or miscarriage if it is occurring before ________ weeks of gestation. A. 14 B. 16 C. 18 D. 20

Choice D is correct. A spontaneous abortion or miscarriage occurs before 20 weeks.

e nurse teaches a client scheduled for an upcoming total hip arthroplasty. Which of the following statements by the client would require follow-up? A. "I will need to bathe with chlorhexidine gluconate solution (CHG) the night before surgery to prevent an infection." B. "I will need to deep breathe and cough every 2 hours." C. "I will have to attend physical therapy sessions following my surgery." D. "I will be prescribed an anticoagulant and need to take it with a sip of water on the day of surgery."

Choice D is correct. After the surgery, the client will be prescribed VTE prevention (sequential compression devices, subcutaneous enoxaparin). The client should not take an anticoagulant or antiplatelet for 5-7 days or as directed by the surgeon before the surgery. This would raise the risk for intra- and postoperative hemorrhage. Other medications such as vitamin E, garlic, and aspirin should be avoided because these will increase the risk of bleeding. Choices A, B, and C are incorrect. Infection is a concern following this surgery, and the client will be prescribed to take a bath with CHG the night before. Once the bath has been completed, the client should then sleep on clean linens and not with pets. The client will be educated to deep breathe and cough and to use the incentive spirometer every 2 hours to prevent atelectasis and pneumonia. Aggressive physical therapy is part of the postoperative process. The PT continues after the client has been discharged, and the client must adhere to these appointments. Additional Info A total hip arthroplasty (THA) carries several postoperative complications ✓ To reduce the risk of a surgical site infection, the client is typically prescribed CHG soap so they can perform a CHG bath the night before. A preoperative antibiotic is typically administered within 60 minutes of the incision. ✓ To reduce the risk of pneumonia, the client will be instructed to deep breathe and cough and to use the incentive spirometer every 2 hours. ✓ To reduce the risk of venous thromboembolism, mechanical (sequential compression devices) and chemical (subcutaneous enoxaparin) prophylaxis are prescribed. ✓ To decrease the risk of hemorrhage, serial hemoglobin and hematocrit (H&H) levels may be ordered every six to eight hours for the first 24 hours postoperative.

The nurse is precepting a graduate nurse as they perform resuscitation on an adult with cardiac arrest. Which action by the graduate requires immediate follow-up by the nurse? A. Assesses the client's pulse by palpating the carotid artery. B. Allows for chest recoil after every chest compression. C. Compresses at a depth of 2 inches on the center breastbone. D. Asks for an automatic external defibrillator after one cycle of CPR.

Choice D is correct. An automatic external defibrillator (AED) should be requested immediately upon establishing that the client is in cardiac arrest. Waiting to request an AED could result in the delay of life-saving care. Choices A, B, and C are incorrect. An adult client should have their pulse palpated using the carotid artery. Chest recoil after every compression is essential to ensure optimal perfusion. Chest compressions for an adult should have a depth of two inches and be over the center breastbone. Add'l Info: When performing CPR, the nurse needs to minimize interruptions and focus on providing effective compressions and ventilations ➢ A compression rate of 100-120/minute is desired ➢ An AED should be made available as urgently as possible ➢ Assessing for a pulse should not take more than ten seconds ➢ When obtaining a pulse for an infant, the nurse should assess the brachial artery. For a child and adult, the nurse will use the carotid artery ➢ Immediate family members should be allowed to be present during resuscitation as this has promoted better grieving

You are caring for a newborn born at term. On your assessment. You note that central cyanosis is present and persistent at five minutes after birth. You attach a pulse oximeter to the newborn. When determining whether or not the infant requires supplemental oxygen, you know that the expected oxygen saturation at 5 minutes after birth is: A. 65-70% B. 70-75% C. 75-80% D. 80-85%

Choice D is correct. At five minutes after birth, the expected SpO2 is in the 80-85% range. Regardless of the cyanosis, if the oxygen saturation is within this range, the infant probably does not need supplemental oxygen at this point. The American Heart Association and American Academy of Pediatrics suggest the following table for Target Pre-ductal Oxygen Saturation levels following birth.

The nurse has become aware of the following client situations. The nurse should first follow up with which client? A client A. that had a myocardial infarction two days ago and the troponin is elevated. B. with infective endocarditis that wants to leave against medical advice (AMA). C. that has arterial insufficiency and is reporting leg pain after walking in the hall. D. recovering from cardiac catheterization that has developed atrial fibrillation.

Choice D is correct. Following a cardiac catheterization, the nurse should assess the client closely for any arrhythmias, including atrial fibrillation. This is a significant finding as the client will have an increased risk of thrombosis, which may migrate and cause a cerebrovascular accident (CVA). Choices A, B, and C are incorrect. Following a myocardial infarction, the troponin level may be elevated for up to two weeks. Thus, the troponin would be expected to be elevated for two days following an insult to the myocardium. A client requesting to leave AMA is a priority for the nurse, but it does not prioritize over a physical need such as a condition change of a client. A classic feature of arterial insufficiency is intermittent claudication, pain induced by the exertion of the leg and relieved by rest. This is an expected finding and not a priority. add'l info: Following a cardiac catheterization, the nurse should assess the client closely for: Decreased or absent pulse in the distal extremity on the affected side Cardiac arrhythmias Signs of shock Bleeding from the catheter insertion site Acute kidney injury Stroke The client will be given head-of-the-bed and activity restrictions immediately following this procedure. The nurse should report any abnormalities in vital signs and retroperitoneal bleeding (bruising in the flank area).

The nurse is caring for the following assigned clients. It would be a priority to follow up with a client who A. is being treated with acute glomerulonephritis (AGN) and has periorbital edema. B. discarded their first urine sample upon starting a 24-urine collection. C. is receiving continuous bladder irrigation and reports the need to void. D. just returned from a hemodialysis session and reports dizziness.

Choice D is correct. Hypotension is a complication associated with hemodialysis. Dizziness may explain this finding, and the nurse should immediately intervene because the client risks falling. The hypotension may be caused by too much fluid removed during the dialysis. This is a safety issue, and the nurse should prioritize this client's needs. Choices A, B, and C are incorrect. AGN has a clinical feature of periorbital edema often present in the morning. This is an expected finding and does not require follow-up. During a 24-hour urine collection, it is appropriate for the first urine to be discarded, which marks the start of the 24-hour collection. During continuous bladder irrigation, a large catheter is utilized and the client reporting that they need to urinate despite having an indwelling catheter is a normal finding not requiring follow-up. Additional Info Hypotension is a common occurrence after hemodialysis and needs to be recognized because the client is at risk for falls and injuries. The most common reason an individual develops hypotension after dialysis is that too much fluid was removed. The nurse should ensure client safety by establishing fall precautions and notifying the provider.

During the early stages of labor, a G3P2 client informs the labor nurse: "I would like to breastfeed, but my breasts became so engorged last time, I could not take it. Do I have to go through that again?" Which of the following responses by the nurse is most appropriate? A. "Keeping your baby on a routine feeding schedule occurring every four hours will help slow the milk production and lessen the engorgement you previously experienced." B. "You can feed your baby formula milk until your milk comes in. This will reduce stimulation and prevent engorgement." C. "Speak to your health care provider (HCP) regarding taking bromocriptine to prevent or suppress your milk production to prevent engorgement." D. "You need to feed your baby as soon as possible. Also, feeding your baby frequently will help prevent breast engorgement."

Choice D is correct. Immediate and frequent breastfeeding is the key to decreasing breast engorgement in breastfeeding women. Also, the initial step in treating breast engorgement is encouraging the mother to immediately breastfeed following birth and continue to do so every two hours. Choice A is incorrect. Feeding and emptying the breasts every four hours increases the risk of engorgement, as does adhering to a strict feeding schedule. The amount of milk capable of being stored in the breasts without causing engorgement varies from person to person. Therefore, following a fixed schedule of feeding/expression may predispose the client to breast engorgement and mastitis due to inadequate milk drainage. Choice B is incorrect. Substituting feedings with formula significantly increases the risk of breast engorgement. Choice C is incorrect. This statement by the nurse is entirely inaccurate and contains misinformation. The medication's previous indication for the prevention of postpartum lactation was withdrawn voluntarily by the manufacturer due to safety issues in 1994. Severe and life-threatening adverse events such as hypertension, myocardial infarction, psychosis, seizures, and stroke had been (and continue to be) reported in postpartum clients when used to inhibit lactation. Bromocriptine is not a preferred medication when treatment is needed for use in breastfeeding clients with an overabundant milk supply (also known as hypergalactia).

The infection control nurse is responding to an outbreak of norovirus in the facility. The nurse should recommend that A. staff wears a surgical mask when providing client care. B. disposable utensils and dishware are used for meals. C. dietary staff wears a face shield when preparing client meals. D. commonly touched surfaces be disinfected with a bleach solution.

Choice D is correct. Norovirus is a virus that is commonly implicated in gastroenteritis. The transmission of norovirus primarily occurs through surfaces contaminated with norovirus and then touching the mouth or other food items. Further, norovirus may be transmitted by having direct contact with another person who is infected or eating food or drinking liquids that are contaminated with norovirus. The most effective disinfectant for norovirus is bleach. It is also recommended that hand hygiene be performed with soap and water as it is superior to alcohol-based hand rubs. Choices A, B, and C are incorrect. Norovirus requires contact precautions for a minimum of 48 hours after the symptoms resolve. A surgical mask is unnecessary to prevent norovirus transmission because infected respiratory droplets do not spread the virus. Further, a face shield is not necessary because a face shield is only worn if splashes of blood or bodily floods are anticipated. Individuals with norovirus should not be preparing meals for others. The PPE required for norovirus is a gown and gloves. Disposable dishes are not necessary for norovirus outbreaks. Silverware and regular dishware may be used as standard detergents effectively eliminate the virus. Add'l Info Norovirus is the most common cause of gastroenteritis ✓ Inadequate hand hygiene is directly implicated in the transmission of this virus ✓ Manifestations may last for 48-72 hours and include nausea and vomiting, watery diarrhea, and abdominal pain ✓ The most significant complication associated with norovirus gastroenteritis is the client developing dehydration (increased temperature, heart rate, and decreased blood pressure) ✓ Infection control is a significant priority because of the high transmission rate Contact precautions are used until the client has been symptom-free for 48 hours Soap and water is the preferred way to sanitize hands Bleach solution should be used to disinfect contaminated surfaces Surfaces should be disinfected starting from the areas with a lower likelihood of norovirus contamination (e.g., tray tables, countertops) to areas with highly contaminated surfaces (e.g., toilets, bathroom fixtures) The most effective way to stop norovirus is through di

The nurse is supervising a new nurse caring for an elderly client. Which of the following statements regarding sensory changes in an older adult, if made by the new nurse, would require follow-up? Select all that apply. "Older adults have an increased acuity for high-pitched tones." "Older adults have a decreased sensitivity to glare." "Older adults have an increased tympanic membrane flexibility." "Older adults have a diminished sound discrimination." "Older adults have a decreased taste sensation."

Choices A, B, & C are correct. These statements are incorrect regarding changes in the older adult and, therefore, require follow-up by the supervising nurse. Older adults commonly experience a loss of acuity for high-pitched frequencies ( presbycusis) due to changes in the inner ear, such as sclerosis ( Choice A). Glare sensitivity is increased, not decreased. As adults age, changes in the eye, such as smaller pupils and reduced light accommodation, can result in increased sensitivity to glare ( Choice B). Age-related changes in the ear also include a thickening of the tympanic membrane rather than increased flexibility ( Choice C). Choice D & E are incorrect. Sound discrimination is altered in the aging adult, which makes it difficult to hear voices in areas with background noise, such as a television ( Choice D). A decrease in the number of taste buds often causes older clients to have difficulty distinguishing between sweet, sour, and bitter tastes ( Choice E). These answers describe correct changes in the older adult and, therefore, do not require follow-up by the supervising nurse.

The nurse is working in the neonatal nursery. Which of the following integumentary assessments in the newborn are normal? Lanugo Milia Mongolian spots Vernix caseosa Mottling

Choices A, B, C, D, and E are all correct. A is correct. Lanugo is fine, soft hair that covers the body and limbs. This is a common finding in newborns and is considered normal. B is correct. Milia are small white bumps typically found on the noses and cheeks of newborns. They are very common, considered normal, and usually go away on their own. C is correct. Mongolian spots are usual in newborns. They are a type of birthmark due to the extra pigment in certain parts of the skin. D is correct. Vernix caseosa is the "cheese-like" coating that covers the skin of a newborn immediately after birth. This is a normal finding and should not be removed from the baby until their first bath, as it provides moisture to their skin. E is correct. A pattern of reddish or purplish discoloration on the skin caused by changes in blood flow. It is a common finding in newborns and is often more noticeable when the baby is cold.

The nurse is caring for a client immediately following an ultrasound-guided thoracentesis. Which client finding requires follow-up? Select all that apply. Nagging cough Trachea slanted more to the unaffected side Rapid heart rate Localized discomfort at the needle site Crackling sound made at the insertion site when palpated

Choices A, B, C, and E are correct. Complications following a thoracentesis include pneumothorax, bleeding, and infection. The biggest concern is pneumothorax. Following a thoracentesis, the client should be assessed for increased respiratory rate, elevated heart rate, nagging cough, decreased oxygen saturation, decreased breath sounds, and air that makes a popping sound near the insertion site when palpated. These are complications and warrant an immediate chest radiograph and supportive measures. Choice D is incorrect. Localized pain at the insertion site is expected. Lidocaine is commonly used prior to the insertion of the catheter. However, some clients may have minor discomfort following this procedure.

The nurse is planning a staff development conference about restraints. Which of the following information should the nurse include? Select all that apply Mittens are not restraints if untethered and the client is physically able to remove the mitt. Elbow restraints may allow a client to remove abdominal or urinary medical devices. Soft wrist restraints should be removed one at a time if a client is violent. Belt restraints may be prescribed on an as-needed basis. Belt restraints should be applied over a client's clothing garments.

Choices A, B, C, and E are correct. These statements are true and should be included in the conference. Mittens are not considered restraints if they are untethered, and the client may be able to remove the mitt. Elbow restraints make removing a medical device near the face or neck difficult. It does not impede the removal of abdominal or urinary medical devices. Belt restraints should be applied over a client's clothing and secured to the bedframe. Significant impairment to a client's skin may result if the belt is directly applied over the skin. Restraints must be removed every two hours to allow for a range of motion. However, if the client is violent, each restraint should be removed once at a time versus both, which allows the client to elope or inflict further violence. Choice D is incorrect. No matter the type, physical restraints cannot be utilized on an as-needed (PRN) basis. Physical restraint usage should be appropriately justified based on the client's behavior during the assessment. This information should not be included because it is incorrect.

The nurse is reviewing a newly hired nurse's understanding of sterile technique. Which statement, if made by the newly hired nurse, would indicate effective understanding? Select all that apply. "I should open sterile packages away from my body." "If the sterile field gets contaminated, I should dispose of everything and start over." "One inch (2.5 cm) border around a sterile drape can be touched with clean fingers." "I should apply sterile gloves on my non-dominant hand first." "An object placed below my waist is considered contaminated."

Choices A, B, C, and E are correct. To prevent contamination, packages should be opened away from the body (reaching over the sterile field causes contamination). If anything is contaminated, it should be removed immediately and discarded. Contaminated objects should be kept away from the sterile field and the sterile field (and its objects) should be kept above the waist. The nurse should avoid touching the sterile field as much as possible but is permitted to touch the outer one-inch (2.5 cm) edge of the field, if necessary. Choice D is incorrect. The correct technique for applying sterile gloves is to apply the glove to the dominant hand first. Using the non-dominant hand, pick up the glove for the dominant hand by touching only the "inside" of the glove's cuff and then applying it to the dominant hand.

The nurse is caring for a 1-day old newborn client diagnosed with jaundice. Which of the following statements is true regarding jaundice in newborns? Select all that apply. Newborn jaundice is often caused by the baby's liver being immature. Phototherapy is a common treatment for newborn jaundice. Breastfeeding should be stopped if a newborn has jaundice. A jaundiced newborn's skin will appear yellow, starting with the face and progressing downwards. Severe, untreated newborn jaundice can lead to kernicterus, a type of brain damage.

Choices A, B, D, and E are correct. A is correct. Newborns often have jaundice due to an immature liver that cannot efficiently process bilirubin, a byproduct of the breakdown of red blood cells. This leads to the accumulation of bilirubin in the blood, causing the yellowish discoloration of the skin and whites of the eyes that is characteristic of jaundice (Maternal / Newborn Foundations of Maternal-Newborn & Women's Health Nursing, 7th Edition). B is correct. Phototherapy, or light therapy, is a standard treatment for newborn jaundice. The baby's skin absorbs the light, which changes the structure of the bilirubin molecules in a way that allows them to be excreted in the urine and stool (Maternal / Newborn Foundations of Maternal-Newborn & Women's Health Nursing, 7th Edition). D is correct. This is a characteristic presentation of newborn jaundice. The yellowish discoloration typically starts on the face. Then it spreads downwards to the chest, abdomen, arms, and legs as the bilirubin level increases (Maternal / Newborn Foundations of Maternal-Newborn & Women's Health Nursing, 7th Edition). E is correct. This is a critical point. If severe jaundice in a newborn is not treated, the high levels of bilirubin can lead to a condition called kernicterus, a form of brain damage that can result in irreversible neurological impairment (Maternal / Newborn Foundations of Maternal-Newborn & Women's Health Nursing, 7th Edition). Choice C is incorrect. C is incorrect. This is incorrect. Breastfeeding should not be stopped if a newborn has jaundice. Frequent feedings can help reduce the bilirubin level in the baby's body by promoting more frequent bowel movements, which helps to eliminate bilirubin from the body. However, in some cases, a temporary switch to formula may be recommended if the baby's bilirubin levels are very high (Maternal / Newborn Foundations of Maternal-Newborn & Women's Health Nursing, 7th Edition).

The nurse is working with a client diagnosed with Diptheria. Which of the following statements are true regarding infection with Diptheria? Select all that apply. The agent of infection is Corynebacterium diphtheriae. The incubation period for this infection is 2 to 5 days. Transmission of the cutaneous diphtheria is via direct contact with the infected person. The communicable period for diphtheria is 3-6 days. Diphtheria is caused by a virus and is highly contagious.

Choices A, B, and C are correct. A is correct. The agent of infection is Corynebacterium diphtheriae. B is correct. The incubation period for diphtheria is 2 to 5 days. C is correct. Diphtheria can manifest in cutaneous or pharyngeal forms. Large droplets can spread pharyngeal diphtheria, so one must use droplet precautions. Transmission of cutaneous diphtheria is via direct contact with the skin sores of an infected person, carriers, or contaminated articles. Choice D is incorrect. The communicable period refers to the duration for which the person can remain infectious. The communicable period for diphtheria varies but is usually 2 to 4 weeks. Choice E is incorrect. Diphtheria is a serious bacterial infection that primarily affects the respiratory system and is spread through respiratory droplets or by contact with contaminated objects. It is not caused by a virus. Add'l info: Diphtheria is a serious bacterial infection that is caused by the bacterium Corynebacterium diphtheriae. It is spread through respiratory droplets or contact with skin lesions of infected individuals. The bacterium produces a toxin that can cause a thick, gray coating in the throat and nose, making it difficult to breathe and swallow. The toxin can also damage the heart, nervous system, and other organs. ✓Vaccination against diphtheria is recommended for all children, and booster shots are recommended every 10 years for adults. Treatment for diphtheria typically involves antibiotics to kill the bacteria and antitoxin to neutralize the toxin. Supportive care, such as respiratory support and hydration, may also be necessary.

The nurse provides oral care to clients in the ICU. What are the benefits of providing oral care to a client in critical care? Select all that apply. It promotes the patient's sense of well-being. It prevents deterioration of the oral cavity. It decreases the incidence of aspiration pneumonia. It eliminates the need for regular flossing. It decreases oropharyngeal secretions. It compensates for an inadequate diet.

Choices A, B, and C are correct. Adequate oral hygiene is essential for promoting a client's sense of well-being and preventing deterioration of the oral cavity. Diligent oral hygiene care can also improve oral health and limit the growth of pathogens in oropharyngeal secretions, decreasing the incidence of both aspiration pneumonia and ventilator associate pneumonia (VAP). Choices D, E, and F are incorrect. Oral care does not eliminate the need for flossing. It can increase saliva production and would not be expected to decrease oropharyngeal secretions. Mouth care has been found to improve appetite but does not compensate for poor nutrition. Additional Info: Adequate oral hygiene helps clear the airway, reduces infection risk, and supports client comfort and self-esteem. When performing oral care, it is important to use a sponge cleaner or soft-bristled toothbrush with water. Glycerin swabs and alcohol-containing mouthwash products can alter the mouth's pH and dry out the mucous membranes, leading to increased bacterial growth. If ordered, diluted hydrogen peroxide solutions can help address crusted areas.

The nurse is caring for a client with nephrotic syndrome. Which of the following assessment findings would be expected? Select all that apply. Proteinuria Hypoalbuminemia Edema Hyperglycemia Jaundice

Choices A, B, and C are correct. Nephrotic syndrome is a kidney disorder. There is renal glomerular damage, which leads to massive proteinuria. Proteinuria is the increased amount of protein in the urine due to protein loss from the bloodstream. Because protein from the bloodstream is being lost in the urine, there is decreased protein in the bloodstream. This is can be referred to as hypoproteinemia, or hypoalbuminemia, as albumin is the type of protein lost in the bloodstream. This hypoalbuminemia causes decreased oncotic pressure in the vasculature, causing profound edema. Proteinuria is the first classic manifestation of nephrotic syndrome (Choice A). Hypoalbuminemia is the second classic manifestation of nephrotic syndrome (Choice B). Edema is the third classic manifestation of nephrotic syndrome (Choice C). Choices D and E are incorrect. Hyperglycemia (blood glucose > 250 mg/dL) is not a feature of nephrotic syndrome, as this would be found in conditions such as pheochromocytoma or Cushing's syndrome. Jaundice would indicate potential dysfunction of the liver and is unrelated to nephrotic syndrome.

While participating in interdisciplinary rounds on the Mother-Baby floor, the provider mentions that your 2-day old client is at risk for phenylketonuria (PKU). The nurse knows that which of the following statements are true regarding this condition? Select all that apply. It is a genetic disorder that is autosomal dominant. Children with phenylketonuria commonly have a musty odor to their urine Hypopigmentation of the hair, skin, and irises is a prominent sign of the disorder. All 50 states require routine screening of newborns for phenylketonuria. Screening for PKU must happen before 24-hours of life

Choices B, C, and D are correct. Phenylketonuria, or PKU, is a genetic disorder that results in central nervous system damage from toxic levels of the essential amino acid phenylalanine. The musty odor, urine smell, and hypopigmentation of the hair, skin, and irises are signs of PKU. It is also true that all 50 states require routine screening of newborns for this disorder. Choice A is incorrect. The disease is inherited in an autosomal recessive manner. Choice E is incorrect. PKU screening should be done after the newborn is 24 hours old. If done prior, PKU can be missed. Additional Info ✓ The PKU newborn screening is done through drops of blood from the newborn's heel onto a special type of paper ✓ PKU symptoms tend to present closer to 6 months of age

The quality improvement nurse plans an initiative to reduce risk factors for falls in the acute care environment. Which of the following risk factors should the nurse recommend be addressed? Select all that apply. inadequate client assessment communication failures dim lighting a client's medical history age of the client

Choices A, B, and C are correct. The nurse should recommend addressing inadequate assessment because this is a significant risk factor for falls that may lead to client injury. The nurse can recommend a standardized fall risk assessment tool and monitor its execution. Communication failures between staff are a significant contributor to falls. The nurse should recommend addressing this by standardizing the handoff report and placing signals outside a client's room (a particular light, fall risk bands, or pictures outside of the client's room). The nurse can plausibly recommend the repair of dim environmental lighting, which is a risk factor for falls, especially when the client is ambulating within their room. Choices D and E are incorrect. The nurse cannot plausibly recommend addressing a client's age or medical history in reducing falls in the acute care environment because the nurse cannot control these variables. While they inform the nurse of a client's risk (older age, medical history of dementia, etc.), they cannot be influenced by the nurse on the quality improvement panel, nor are they related to the environment. Additional Info Risk factors for falls ✓ Previous client fall ✓ A client with altered cognition ✓ Tubes or devices connected to the client ✓ Sensory deficit(s) (poor vision) ✓ Poor fitting footwear ✓ Poor staff communication ✓ Inadequate client assessment ✓ Medications such as benzodiazepines, anticholinergics, diuretics, antihypertensives, and opioids

The nurse is teaching a continuing education course regarding vaccines and pregnancy. It would be appropriate for the nurse to state which vaccines are not recommended to be administered during pregnancy? Select all that apply. Measles, mumps, and rubella (MMR) Varicella Hepatitis A Inactivated Influenza Tdap (Tetanus, Diphtheria, Pertussis) Human papillomavirus (HPV)

Choices A, B, and F are correct. These vaccines are contraindicated during pregnancy. If the client is scheduled to receive any of these vaccines, the nurse should inquire about the client's pregnancy status prior to vaccine administration. Choices C, D, and E are incorrect. Hepatitis A and B are safe to administer during pregnancy. Influenza vaccination is permitted during pregnancy but cannot be the live influenza vaccine (LAIV). The inactivated influenza vaccine is safe and approved for individuals who are pregnant. Tdap (Tetanus, Diphtheria, Pertussis) is permitted during pregnancy. Additional Info Vaccines either not recommended or contraindicated during pregnancy include: ✓ MMR ✓ Varicella ✓ Zoster ✓ HPV ✓ Polio ✓ Any live vaccine

The nurse is preparing a class for antepartum clients on fetal development. Which of the following statements is true regarding fetal circulation? Select all that apply. There are high pressures in the fetal lungs causing decreased pulmonary circulation. Blood shunts from left to right in the fetal circulation. The ductus venosus allows freshly oxygenated blood to go to the fetal brain first. There are higher pressures in the right atrium in the fetal circulation. After birth, the fetal circulation system undergoes significant changes as the baby begins to breathe and the lungs become functional.

Choices A, C, D and E are correct. A is correct. In fetal circulation, the alveoli are filled with fluid. This causes high pressures in the fetal lungs, which shunts blood away from the pulmonary circulation. C is correct. The ductus venosus is a bypass in fetal circulation that shunts blood away from the weak fetal liver and to the brain. This allows the brain to get fresh oxygen first. D is correct. The pressures on the right side of the heart are higher in fetal circulation than on the left side of the heart. E is correct. The foramen ovale and ductus arteriosus normally close shortly after birth, allowing the baby's lungs to take over oxygenation of the blood. Choice B is incorrect. Blood shunts from right to left in the fetal circulation; this is due to increased pulmonary pressures caused by the fluid-filled alveoli. The high pulmonary pressures increase pressure on the right side of the heart, creating a gradient across the foramen ovale shunting blood from right to left.

The nurse is teaching progressive relaxation techniques to a client. Which of the following statements by the client indicates that the teaching has been effective? Select all that apply. "I will breathe in and out in rhythm." "I expect my pulse to be faster afterwards." "I expect to require less pain medication." "I expect my muscles to feel less tense." "I will report any increased sensitivity."

Choices A, C, D, and E are correct. Progressive relaxation involves rhythmic breathing and progressive tension and relaxation of one muscle group at a time. When implemented, clients typically experience decreased muscle tension and a reduction in the need for pharmacologic measures to relieve pain and anxiety. Although sensitivity may be normal for clients who are new to progressive relaxation exercises, any sensitivity or exhaustion should be reported and monitored so the nurse can decide whether the client would be better suited for passive relaxation techniques. Choice B is incorrect. When relaxation techniques are properly implemented, the client should experience a decreased pulse rate.

Which forms of nonverbal communication can be viewed differently among members of different and diverse cultures? Select all that apply. Silence A smile Eye contact Touch Bodily posture

Choices A, C, D, and E are correct. Silence, eye contact, touch, and bodily posture are all forms of nonverbal communication that can be viewed and perceived differently among members of different and diverse cultures. Some cultures can see silence to be a lack of attention, while others can perceive silence as a compassionate way that understanding is conveyed. Some view eye contact as aggressive and hostile while other cultures see eye contact as connectedness with others. Some cultures perceive touch as inappropriate and invasive while others recognize touch as a sign of caring and compassion. Lastly, many bodily postures and gestures differ significantly among various cultures. A smile is a relatively universal sign of joy and happiness. Choice B is incorrect. A smile is a relatively universal sign of joy and happiness among all, if not most, cultures. Therefore this is the incorrect answer to the question.

The nurse is caring for a client with atrial fibrillation who takes prescribed warfarin. Which alternative therapies should the nurse advise this patient to avoid? Select all that apply. Ginger root Aloe vera Garlic Ginko biloba Saw palmetto

Choices A, C, D, and E are correct. The client taking prescribed warfarin should avoid alternative therapies that may potentiate the anticoagulant effects and increase bleeding risk. Alternative therapies such as Ginkgo Biloba, ginger root, garlic, and saw palmetto increase the bleeding risk in a client taking warfarin. The client should be advised against taking these medications. Choice B is incorrect. Aloe vera is used in alternative/complementary medicine purported to relieve constipation. Aloe may cause electrolyte imbalances and decreased blood glucose levels but is not known to increase bleeding risk.

When educating an adolescent diagnosed with bacterial conjunctivitis about how to prevent the spread of their infection, which of the following points should you include? Select all that apply. Do not share towels or washcloths with family members. Stay home from school until they have taken antibiotics for 48 hours. Apply a warm compress to lessen any irritation. Throw out the contact lenses and get new ones. Perform hand hygiene, especially prior to touching face or eyes

Choices A, D and E are correct. To prevent the spread of bacterial conjunctivitis, it is essential not to share towels or washcloths with anyone while infected (Choice A). This is appropriate advice for preventing reinfection of bacterial conjunctivitis. If a client wears the same contact lenses, they will likely spread the disease for a second time and become infected again. They should also be instructed to discard their eye makeup to prevent reinfection (Choice D). Remind adolescent about how to perform proper hand hygiene and to perform prior to touching face or eyes to prevent spread of bacteria (Choice E.) Choice B is incorrect. It is okay to go back to school after 24 hours of antibiotics, not 48 hours. Choice C is incorrect. Applying a compress will help improve comfort for the client with bacterial conjunctivitis, but has nothing to do with preventing the spread of the infection. Furthermore, utilizing a warm compress would cause further irritation with bacterial conjunctivitis; instead, a cold compress should be used. add'L info Bacterial conjunctivitis is commonly called "pink eye" due to the pinkness/redness it causes is one or both eyes ✓ Other symptoms may include itchy eyes, discharge, light sensitivity, gritty feeling, and watery eyes

The nurse is participating on a committee changing the hospital safety and security plan. Which of the following statements by the nurse would be appropriate to make? Select all that apply. Open visitation should be implemented in the newborn nursery. Visitors should always wear a badge while in the hospital Oral temperatures should be obtained for all visitors Hand sanitizing stations should be offered throughout the facility Disaster drills should be conducted to ensure staff competency

Choices B and E are correct. Proper visitor identification is essential in keeping a hospital secure. This allows for rapid identification and a log of all visitors within the facility. Disaster drills should be conducted to ensure that staff is competent with procedures related to certain threats such as mass bioterrorism, active shooter training, or fire. Choices A, C, and D are incorrect. Open visitation should not be implemented in a newborn nursery because of the risk of infant abduction. Visitors may learn the hospital's layout and certain procedures, enabling a more inconspicuous abduction. Checking the oral temperatures of visitors and providing hand sanitizing stations is essential in infection prevention, but they are not pertinent to the hospital's safety and security plan.

The registered nurse (RN) assigns client care to a licensed practical/vocational nurse (LPN/VN). Which of the following should the RN assign to the LPN? Select all that apply. A client requiring an assessment of their current medications A client needing a nasogastric tube (NGT) for enteral feedings A client with an insulin pump and is unsure of how to load the insulin A client with unstable blood pressure following adrenalectomy. A client requiring airborne isolation and bronchodilators via an inhaler

Choices B and E are correct. Skills such as the insertion of an NGT are within the scope of an LPN/VN. The RN can delegate this to the LPN/VN. Further, LPN/VN's may care for a client in isolation as well as administer bronchodilators via an inhaler. Practical/vocational nurses should get the most stable patient assignment. Choices A, C, and D are incorrect. Assessment of a client's medication regimen is the responsibility of the RN, as well as teaching a client who is unsure about their insulin pump. Finally, the LPN should be assigned the most stable client. A client following adrenalectomy may consequently develop unstable blood pressure; thus, this should be assigned to the RN.

Which interventions are appropriate for venous thromboembolism prophylaxis when caring for a non-ambulatory client? Select all that apply. Floating both of the heels using a pillow Apply sequential compression devices to the lower extremities Encourage range of motion exercises in the lower extremities Apply compression hose to the lower extremities Administer enoxaparin subcutaneously, as prescribed

Choices B, C, D, and E are correct. Sequential compression devices (SCDs) provide DVT prophylaxis by applying intermittent external pressure, pushing blood into deep veins, reducing stasis, and improving venous return. Range of motion exercises should be encouraged for a non-ambulatory client to encourage venous return. Thromboembolic deterrent (TED) hose promotes venous blood flow, prevents venous dilation, improves venous valve function, and stimulates endothelial fibrinolytic activity. Enoxaparin is a low molecular weight-based heparin given subcutaneously in the abdomen. This is a form of chemical VTE (venous thromboembolism) prophylaxis. Choice A is incorrect. Floating the heels is a measure to prevent a pressure ulcer. The heels may be floated using pillows or a specialty mattress that relieves the pressure. This would not be a measure to prevent a VTE. NCLEX Category: Reduction of Risk Potential

A nurse is caring for a group of preoperative clients. Which client situation requires follow-up? A client Select all that apply. stating that they took their prescribed carbamazepine with a sip of water. receiving dextrose 5% in water (D5W) and has a blood glucose of 266 mg/dL. reporting that they shaved their abdomen for their scheduled appendectomy. reporting difficulty with their last surgery, stating they got 'a really high fever'. reporting burning upon urination and increased urinary frequency.

Choices B, C, D, and E are correct. These clients require follow-up. Preoperative (and postoperative) hyperglycemia is detrimental to optimal outcomes. This client has a glucose of 266 mg/dL, which is hyperglycemia. This client should also have the prescribed infusion of D5W questioned, as this solution would further increase the glucose. Clipping hair at the operative site is the best practice because it reduces the risk of surgical site infection. If shaving has to be done, it is completed immediately before the incision to reduce the chance of postoperative infection. The client stated that they got a high fever after their previous surgery and requires follow-up. This could be a concern for malignant hyperthermia. Although rare, this genetic disorder can be life-threatening when the client is exposed to certain anesthesia. The client reported burning upon urination preoperative and requires follow-up as preoperative infections may cause surgery cancelation as they complicate healing. Choice A is incorrect. This client does not require follow-up. Carbamazepine is an anticonvulsant and is commonly permitted to be taken with a sip of water to prevent seizure activity. Add'l info When performing a preoperative surgical assessment, the nurse assesses the client's physical status and reviews elements such as Adherence to nothing by mouth (NPO) status Preoperative laboratory and diagnostic data Basic understanding of the procedure Discharge planning Postoperative education

The nurse is caring for a cancer patient who is receiving chemotherapy. The patient is experiencing weight loss as a result of intermittent nausea. The nurse should implement which of the following nursing interventions to help with the patient's nausea? Select all that apply. Suggest using hot sauce and strong herbs for bland foods. Serve small meals every 2-3 hours. Provide meals that are best eaten at room temperature. Encourage the patient to brush their teeth in the afternoon rather than in the morning. Serve high-fat and protein dense foods.

Choices B, C, and D are correct. Serving small meals every 2-3 hours may help keep nausea at bay. Food eaten at room temperature and delaying teeth brushing till the afternoon may also improve nausea. Choice A is incorrect. Spicy foods and healthy herbs may heighten nausea in the patient receiving chemotherapy. Choice E is incorrect. High-fat foods are especially nauseating for those experiencing nausea.

The nurse is preparing to provide care for a client with disseminated herpes zoster. The nurse plans to don which personal protective equipment (PPE)? Select all that apply. goggles gown gloves shoe covers n95 respirator Surgical face mask

Choices B, C, and E are correct. When varicella zoster is disseminated, it can be transmitted through airborne means and by direct contact with the lesions. The isolation required is contact + airborne. This means the nurse should wear an N95 respirator, high-efficiency particulate air filter respirator, gown, and gloves. Herpes zoster, also known as shingles, is caused by the reactivation of the varicella-zoster virus (VZV), the same virus that causes varicella (chickenpox). Primary infection with VZV causes varicella. Once the illness resolves, the virus remains latent in the dorsal root ganglia. VZV can be reactive later in a person's life and create a painful, maculopapular rash called herpes zoster. When herpes zoster (varicella-zoster) (shingles) is localized and can be covered, standard precautions are implemented until all of the lesions have crusted over. When herpes zoster (varicella-zoster) (shingles) is disseminated, airborne + contact precautions are implemented for the duration of the illness. Choices A and D are incorrect. Goggles and shoe covers are not needed for airborne or contact precautions. Choice F is incorrect. A surgical face mask filters only large particles and will not protect against herpes zoster.

The nurse is caring for a client scheduled for an amniocentesis. Which of the following statements would require follow-up? Select all that apply. "This test may tell me the gender of my baby." "I will receive intravenous (IV) sedation for this test." "I may have cramping after this procedure." "I may be given a medicine to stimulate contractions." "The results will tell me how my baby will handle labor."

Choices B, D, and E are correct. These statements are false and require follow-up. Amniocentesis is an ultrasound-guided test used in the detection of fetal abnormalities. Under ultrasound guidance, a thin needle is inserted into the amniotic sac to remove a sample of amniotic fluid. The fluid is then sent for analysis. Amniocentesis does not require intravenous sedation (choice B). The client will not be given medication to stimulate contractions for the amniocentesis procedure (choice D). Ureterotonic drugs (oxytocin challenge) are administered in a contraction stress test (CST), not amniocentesis. Once again, a CST will determine how the fetus will handle labor, not an amniocentesis (choice E). These statements require follow-up counseling and education to correct the client's understanding.

The nurse observes a student inserting an indwelling urinary catheter into a female client. Which action by the student requires follow-up by the nurse? The student (Select all that apply) applies clean gloves to clean the perineal area with soap and water. asks the client to bear down gently and slowly insert the catheter through the urethral meatus. separates the labia with the fingers of the dominant hand when cleaning with antiseptic solution. secures the catheter tubing to the inner thigh. attaches the drainage bag to the side rails of bed.

Choices C and E are correct. These actions by the student are incorrect and require follow-up by the nurse. Clean gloves are used to clean the perineum, not the labia. When cleaning the labia with the antiseptic solution, the student should wear sterile gloves. The student should first separate the labia with the fingers of the nondominant hand to fully expose the urethral meatus. Once the non-dominant/nonsterile hand touches the patient, it is now considered contaminated (non-sterile). Therefore, the nonsterile nondominant hand is held in this position throughout the procedure until the catheter is inserted. While holding the labia apart with the nondominant non-sterile hand, the student nurse should clean the labia with the antimicrobial solution using their sterile dominant hand. Acting correctly will greatly decrease the risk of contamination. Following this, the student should pick up the lubricated catheter with the sterile dominant hand, place the distal end of the catheter in the receptacle, and steadily insert the catheter into the urethral meatus. The drainage bag of the urinary catheter should not be secured to the bed's side rails because it will move with the side rails and cause tension on the tubing that may cause urinary trauma. The bag should be secured to the bed frame, not the side rails.

The nurse is educating a group of nursing students about complications during pregnancy. Which of the following statements is true regarding the premature rupture of membranes (PROM)? "PROM is when the membranes rupture before 37 weeks gestation." "In a normal delivery, membranes are expected to rupture before labor begins." "A priority nursing intervention with PROM is to monitor for infection." "When observing the fluid after the rupture of membranes, it should be clear and without odor." "PROM may lead to preterm birth if before 37 weeks

Choices C, D, and E are correct. C is correct. A priority nursing intervention with PROM is to monitor for infection. One of the most critical observations you must make is of the color, odor, consistency, and amount of the amniotic fluid when the rupture of membranes occurs. Any discolored or malodorous fluid may indicate an infection. After the breakdown of membranes occurs, the nurse should monitor the mother's temperature, WBC count, CRP, and other disease markers. D is correct. It is essential to assess the color, odor, consistency, and amount of fluid when the rupture of membranes occurs. If the liquid is green or yellow and rancid, it indicates infection. If the fluid is brown or black, it means meconium passing in utero. The expected finding of amniotic fluid is a clear fluid with no odor. E is correct. PROM may lead to preterm birth if it occurs before 37 weeks of gestation. If the membranes rupture prematurely and labor does not spontaneously begin, healthcare providers may need to induce labor to prevent infections and other complications. If the membranes rupture before 37 weeks gestation, the correct terminology is PPROM. Choices A and B are incorrect. A is incorrect. PROM stands for premature rupture of membranes. This is defined as the rupture of membranes (or "water breaking") before labor begins. This term is not related to what gestation the membranes rupture. If the membranes rupture before 37 gestational weeks, that is considered preterm premature rupture of membranes (PPROM) B is incorrect. It is not expected that the membranes will rupture before the labor begins. In a standard delivery, membranes rupture after the mother has already started having regular contractions, dilating, and effacing. When the layers separate before labor begins, it is called PROM, or premature rupture of membranes.

The nurse is reviewing test results for an antepartum client. Which of the following antepartum test results indicate a need to further follow up? Select all that apply. Contraction stress test - negative Nonstress test - reactive Contraction stress test - positive Nonstress test - nonreactive Rh-negative blood type

Choices C, D, and E are correct. A positive contraction stress test means the baby had decelerations in response to contractions and therefore, may not tolerate labor. Therefore, follow-up is needed (Choice C). A nonreactive nonstress test means that the baby did not have two or more 15 by 15 accelerations during the 20 minute test period and is not responding appropriately to movement. Follow-up would be needed for this test result, most likely with a contraction stress test (Choice D). If the mother has Rh-negative blood and the father has Rh-positive blood, further testing and treatment may be needed to prevent complications related to Rh incompatibility. (Choice E) Choice A is incorrect. In a contraction stress test, the result we want is negative. This means that the baby did not have decelerations in response to contractions. Follow up would not be needed for this test result. Choice B is incorrect. In a non-stress test, the result we want is reactive. This means that the baby had two or more 15 by 15 accelerations during the 20 minute test period and is responding appropriately to movement. Follow up would not be needed for this test result.

The nurse is caring for a group of assigned clients. Which of the following actions by the nurse is an example of a nurse-initiated intervention? Select all that apply. The nurse administers 1000 mg of ciprofloxacin to a client with pneumonia. The nurse consults with a psychiatrist for a client suspected of pain medication abuse. The nurse checks the skin of bedridden clients for signs of breakdown. A nurse assists an orthodox Jewish client with ordering a kosher meal. The nurse records the intake & output of a client as prescribed by her physician. The nurse provides teaching to a client on how to care for a newly placed ostomy.

Choices C, D, and F are correct. Nurse-initiated interventions, also known as independent nursing actions, involve carrying out nurse-prescribed interventions resulting from their assessment of client needs that are written on the nursing care plan, as well as other activities that nurses can initiate without the direction or supervision of another healthcare personnel. The nurse can take initiative independently by monitoring clients' skin for breakdown, assisting a client to order an appropriate meal, and providing education to clients and family members. Choices A, B, and E are incorrect. Administration of medications and initiation of intake-output monitoring are dependent interventions because these actions require a physician's order or physician supervision. Consulting with a psychiatrist is a collaborative intervention, not an independent nursing action.

he nurse is planning a staff development conference about pain management. Which statement would be appropriate to include? Select all that apply. Infants do not have developed pain sensors. A lack of behavioral signs of pain negates pain. The amount of pain has a positive correlation with the extent of tissue damage. Self-report is the most reliable method of pain assessment. Analgesics should be administered via the oral route when possible.

Choices D and E are correct. These statements about pain are appropriate to include. Self-report is considered the most accurate indicator in assessing pain. The oral route is the preferred method of analgesic delivery due to being easiest to administer, better tolerated, and more cost-effective than intravenous medication. Choices A, B, and C are incorrect. These are commonly held misconceptions about pain/pain management. A lack of physiological and behavioral signs of pain does not indicate the absence of pain. Research has shown that infants can experience pain and confirmed that pain does not necessarily equate to tissue damage. Clients may experience intense pain with no significant tissue damage or vice versa.

The nurse is caring for a client in active labor 23-year-old primipara at 39 gestational weeks was admitted for induction via oxytocin. Currently, she is 100% effaced and 10 cm dilated. An internal fetal spiral electrode and intrauterine pressure catheter were placed. Uterine contractions are now 2 to 2.5 minutes apart, 70 to 90 seconds in duration. The fetal heart tracing showed decreased fetal heart rate following uterine contraction. This pattern was present in more than 50% of the uterine contractions. med: Oxytocin via continuous infusion Based on the fetal heart rate tracing, the client is experiencing _________ that is caused by____________

Late Deceleration.......reduced blood flow to the placenta The client is experiencing late decelerations because the fetal heart rate showed a decrease following the uterine contraction. This is further supported by this non-reassuring pattern occurring more than 50% of the uterine contractions. Late decelerations are concerning because they it may cause fetal hypoxia. Late decelerations are commonly caused by maternal hypotension, therefore, changing maternal position is key to resolving this non-reassuring pattern. Early decelerations are a reassuring pattern caused by fetal head compression. Early decelerations are symmetric in shape and have a gradual decrease and return of FHR baseline that mirrors a uterine contraction. Variable decelerations are non-reassuring and triggered by umbilical cord compression.


Conjuntos de estudio relacionados

Pharmacology Test 2 (Chapters 3-5)

View Set

Test 3 - Zhang Consumer Behavior Ch 8 - 11

View Set

Ch 22: The Ordeal of Reconstruction (short answer)

View Set

Chapter 57: Drugs Affecting Gastrointestinal Secretions

View Set

Chapters 13-14 - Capacitors & Inductors True or False Questions

View Set

Fundamentals Midterm: Ch. 21, 27, 28, 31, 38, 39, 40, 44, 45, 48, and seizure precautions.

View Set